You are on page 1of 163

Chapter 7

TIME VALUE OF MONEY

1. Value five years hence of a deposit of Rs.1,000 at various interest rates is as follows:

r = 8% FV5 = Rs.1469

r = 10% FV5 = Rs.1611

r = 12% FV5 = Rs.1762

r = 15% FV5 = Rs.2011

2. 30 years

3. In 12 years Rs.1000 grows to Rs.8000 or 8 times. This is 23 times the initial deposit. Hence
doubling takes place in 12 / 3 = 4 years.

According to the Rule of 69, the doubling period is:

0.35 + 69 / Interest rate

Equating this to 4 and solving for interest rate, we get

Interest rate = 18.9%.

4. Saving Rs.2000 a year for 5 years and Rs.3000 a year for 10 years thereafter is equivalent to
saving Rs.2000 a year for 15 years and Rs.1000 a year for the years 6 through 15.
Hence the savings will cumulate to:
2000 x FVIFA (10%, 15 years) + 1000 x FVIFA (10%, 10 years)
= 2000 x 31.772 + 1000 x 15.937 = Rs.79481.

5. Let A be the annual savings.

A x FVIFA (12%, 10 years) = 1,000,000


A x 17.549 = 1,000,000

So, A = 1,000,000 / 17.549 = Rs.56,983.

6. 1,000 x FVIFA (r, 6 years) = 10,000

FVIFA (r, 6 years) = 10,000 / 1000 = 10

1
From the tables we find that
FVIFA (20%, 6 years) = 9.930
FVIFA (24%, 6 years) = 10.980

Using linear interpolation in the interval, we get:

20% + (10.000 9.930)


r= x 4% = 20.3%
(10.980 9.930)

7. 1,000 x FVIF (r, 10 years) = 5,000


FVIF (r,10 years) = 5,000 / 1000 = 5

From the tables we find that

FVIF (16%, 10 years) = 4.411


FVIF (18%, 10 years) = 5.234

Using linear interpolation in the interval, we get:

(5.000 4.411) x 2%
r = 16% + = 17.4%
(5.234 4.411)

8. The present value of Rs.10,000 receivable after 8 years for various discount rates (r ) are:
r = 10% PV = 10,000 x PVIF(r = 10%, 8 years)
= 10,000 x 0.467 = Rs.4,670

r = 12% PV = 10,000 x PVIF (r = 12%, 8 years)


= 10,000 x 0.404 = Rs.4,040

r = 15% PV = 10,000 x PVIF (r = 15%, 8 years)


= 10,000 x 0.327 = Rs.3,270

9. Assuming that it is an ordinary annuity, the present value is:


2,000 x PVIFA (10%, 5years)
= 2,000 x 3.791 = Rs.7,582

10. The present value of an annual pension of Rs.10,000 for 15 years when r = 15% is:
10,000 x PVIFA (15%, 15 years)
= 10,000 x 5.847 = Rs.58,470

2
The alternative is to receive a lumpsum of Rs.50,000.

Obviously, Mr. Jingo will be better off with the annual pension amount of Rs.10,000.

11. The amount that can be withdrawn annually is:


100,000 100,000
A = ------------------ ------------ = ----------- = Rs.10,608
PVIFA (10%, 30 years) 9.427

12. The present value of the income stream is:


1,000 x PVIF (12%, 1 year) + 2,500 x PVIF (12%, 2 years)
+ 5,000 x PVIFA (12%, 8 years) x PVIF(12%, 2 years)

= 1,000 x 0.893 + 2,500 x 0.797 + 5,000 x 4.968 x 0.797 = Rs.22,683.

13. The present value of the income stream is:


2,000 x PVIFA (10%, 5 years) + 3000/0.10 x PVIF (10%, 5 years)
= 2,000 x 3.791 + 3000/0.10 x 0.621
= Rs.26,212

14. To earn an annual income of Rs.5,000 beginning from the end of 15 years from now, if the
deposit earns 10% per year a sum of
Rs.5,000 / 0.10 = Rs.50,000
is required at the end of 14 years. The amount that must be deposited to get this sum is:
Rs.50,000 / PVIF (10%, 14 years) = Rs.50,000 / 3.797 = Rs.13,165

15. Rs.20,000 =- Rs.4,000 x PVIFA (r, 10 years)


PVIFA (r,10 years) = Rs.20,000 / Rs.4,000 = 5.00

From the tables we find that:


PVIFA (15%, 10 years) = 5.019
PVIFA (18%, 10 years) = 4.494

Using linear interpolation we get:


5.019 5.00
r = 15% + ---------------- x 3%
5.019 4.494

= 15.1%

16. PV (Stream A) = Rs.100 x PVIF (12%, 1 year) + Rs.200 x


PVIF (12%, 2 years) + Rs.300 x PVIF(12%, 3 years) + Rs.400 x

3
PVIF (12%, 4 years) + Rs.500 x PVIF (12%, 5 years) +
Rs.600 x PVIF (12%, 6 years) + Rs.700 x PVIF (12%, 7 years) +
Rs.800 x PVIF (12%, 8 years) + Rs.900 x PVIF (12%, 9 years) +
Rs.1,000 x PVIF (12%, 10 years)

= Rs.100 x 0.893 + Rs.200 x 0.797 + Rs.300 x 0.712


+ Rs.400 x 0.636 + Rs.500 x 0.567 + Rs.600 x 0.507
+ Rs.700 x 0.452 + Rs.800 x 0.404 + Rs.900 x 0.361
+ Rs.1,000 x 0.322

= Rs.2590.9

Similarly,
PV (Stream B) = Rs.3,625.2
PV (Stream C) = Rs.2,851.1

17. FV5 = Rs.10,000 [1 + (0.16 / 4)]5x4


= Rs.10,000 (1.04)20
= Rs.10,000 x 2.191
= Rs.21,910

18. FV5 = Rs.5,000 [1+( 0.12/4)] 5x4


= Rs.5,000 (1.03)20
= Rs.5,000 x 1.806
= Rs.9,030

19 A B C

Stated rate (%) 12 24 24

Frequency of compounding 6 times 4 times 12 times

Effective rate (%) (1 + 0.12/6)6- 1 (1+0.24/4)4 1 (1 + 0.24/12)12-1

= 12.6 = 26.2 = 26.8

Difference between the


effective rate and stated
rate (%) 0.6 2.2 2.8

20. Investment required at the end of 8th year to yield an income of Rs.12,000 per year from the
end of 9th year (beginning of 10th year) for ever:

Rs.12,000 x PVIFA(12%, )

4
= Rs.12,000 / 0.12 = Rs.100,000

To have a sum of Rs.100,000 at the end of 8th year , the amount to be deposited now is:
Rs.100,000 Rs.100,000
= = Rs.40,388
PVIF(12%, 8 years) 2.476

21. The interest rate implicit in the offer of Rs.20,000 after 10 years in lieu of Rs.5,000 now is:
Rs.5,000 x FVIF (r,10 years) = Rs.20,000

Rs.20,000
FVIF (r,10 years) = = 4.000
Rs.5,000

From the tables we find that


FVIF (15%, 10 years) = 4.046

This means that the implied interest rate is nearly 15%.


I would choose Rs.20,000 for 10 years from now because I find a return of 15% quite
acceptable.

22. FV10 = Rs.10,000 [1 + (0.10 / 2)]10x2


= Rs.10,000 (1.05)20
= Rs.10,000 x 2.653
= Rs.26,530

If the inflation rate is 8% per year, the value of Rs.26,530 10 years from now, in terms of
the current rupees is:
Rs.26,530 x PVIF (8%,10 years)
= Rs.26,530 x 0.463 = Rs.12,283

23. A constant deposit at the beginning of each year represents an annuity due.
PVIFA of an annuity due is equal to : PVIFA of an ordinary annuity x (1 + r)
To provide a sum of Rs.50,000 at the end of 10 years the annual deposit should
be

Rs.50,000
A = FVIFA(12%, 10 years) x (1.12)

Rs.50,000
= = Rs.2544
17.549 x 1.12

5
24. The discounted value of Rs.20,000 receivable at the beginning of each year from 2005 to
2009, evaluated as at the beginning of 2004 (or end of 2003) is:
Rs.20,000 x PVIFA (12%, 5 years)
= Rs.20,000 x 3.605 = Rs.72,100.

The discounted value of Rs.72,100 evaluated at the end of 2000 is


Rs.72,100 x PVIF (12%, 3 years)
= Rs.72,100 x 0.712 = Rs.51,335

If A is the amount deposited at the end of each year from 1995 to 2000 then
A x FVIFA (12%, 6 years) = Rs.51,335
A x 8.115 = Rs.51,335
A = Rs.51,335 / 8.115 = Rs.6326

25. The discounted value of the annuity of Rs.2000 receivable for 30 years, evaluated as at the
end of 9th year is:
Rs.2,000 x PVIFA (10%, 30 years) = Rs.2,000 x 9.427 = Rs.18,854
The present value of Rs.18,854 is:
Rs.18,854 x PVIF (10%, 9 years)
= Rs.18,854 x 0.424
= Rs.7,994
26. 30 per cent of the pension amount is
0.30 x Rs.600 = Rs.180
Assuming that the monthly interest rate corresponding to an annual interest rate of 12% is
1%, the discounted value of an annuity of Rs.180 receivable at the end of each month for 180
months (15 years) is:
Rs.180 x PVIFA (1%, 180)

(1.01)180 - 1
Rs.180 x ---------------- = Rs.14,998
.01 (1.01)180
If Mr. Ramesh borrows Rs.P today on which the monthly interest rate is 1%

P x (1.01)60 = Rs.14,998
P x 1.817 = Rs.14,998
Rs.14,998
P = ------------ = Rs.8254
1.817

27. Rs.300 x PVIFA(r, 24 months) = Rs.6,000


PVIFA (4%,24) = Rs.6000 / Rs.300 = 20

From the tables we find that:


PVIFA(1%,24) = 21.244

6
PVIFA (2%, 24) = 18.914

Using a linear interpolation


21.244 20.000
r = 1% + ---------------------- x 1%
21.244 18,914

= 1.53%

Thus, the bank charges an interest rate of 1.53% per month.


The corresponding effective rate of interest per annum is
[ (1.0153)12 1 ] x 100 = 20%

28. The discounted value of the debentures to be redeemed between 8 to 10 years evaluated at
the end of the 5th year is:
Rs.10 million x PVIF (8%, 3 years)
+ Rs.10 million x PVIF (8%, 4 years)
+ Rs.10 million x PVIF (8%, 5 years)

= Rs.10 million (0.794 + 0.735 + 0.681)


= Rs.2.21 million

If A is the annual deposit to be made in the sinking fund for the years 1 to 5,
then
A x FVIFA (8%, 5 years) = Rs.2.21 million
A x 5.867 = Rs.2.21 million
A = 5.867 = Rs.2.21 million
A = Rs.2.21 million / 5.867 = Rs.0.377 million

29. Let `n be the number of years for which a sum of Rs.20,000 can be withdrawn annually.

Rs.20,000 x PVIFA (10%, n) = Rs.100,000


PVIFA (15%, n) = Rs.100,000 / Rs.20,000 = 5.000

From the tables we find that


PVIFA (10%, 7 years) = 4.868
PVIFA (10%, 8 years) = 5.335

Thus n is between 7 and 8. Using a linear interpolation we get

5.000 4.868
n=7+ ----------------- x 1 = 7.3 years
5.335 4.868

7
30. Equated annual installment = 500000 / PVIFA(14%,4)
= 500000 / 2.914
= Rs.171,585

Loan Amortisation Schedule

Beginning Annual Principal Remaining


Year amount installment Interest repaid balance
------ ------------- --------------- ----------- ------------- -------------
1 500000 171585 70000 101585 398415
2 398415 171585 55778 115807282608
3 282608 171585 39565 132020 150588
4 150588 171585 21082 150503 85*

(*) rounding off error

31. Define n as the maturity period of the loan. The value of n can be obtained from the
equation.

200,000 x PVIFA(13%, n) = 1,500,000


PVIFA (13%, n) = 7.500

From the tables or otherwise it can be verified that PVIFA(13,30) = 7.500


Hence the maturity period of the loan is 30 years.

32. Expected value of iron ore mined during year 1 = Rs.300 million

Expected present value of the iron ore that can be mined over the next 15 years assuming a
price escalation of 6% per annum in the price per tonne of iron

1 (1 + g)n / (1 + i)n
= Rs.300 million x ------------------------
i-g

= Rs.300 million x 1 (1.06)15 / (1.16)15


0.16 0.06

= Rs.300 million x (0.74135 / 0.10)


= Rs.2224 million

8
MINICASE

Solution:

1. How much money would Ramesh need 15 years from now?

500,000 x PVIFA (10%, 15years)


+ 1,000,000 x PVIF (10%, 15years)
= 500,000 x 7.606 + 1,000,000 x 0.239
= 3,803,000 x 239,000
= Rs.4,042,000

2. How much money should Ramesh save each year for the next 15 years to be able to meet his
investment objective?

Rameshs current capital of Rs.600,000 will grow to :

600,000 (1.10)15 = 600,000 x 4.177 = Rs 2,506,200

This means that his savings in the next 15 years must grow to :

4,042,000 2,506,200 = Rs 1,535,800

So, the annual savings must be :


1,535,800 1,535,800
= = Rs.48,338
FVIFA (10%, 15 years) 31.772

3. How much money would Ramesh need when he reaches the age of 60 to meet his donation
objective?

200,000 x PVIFA (10% , 3yrs) x PVIF (10%, 11yrs)

= 200,000 x 2.487 x 0.317 = 157,676

4. What is the present value of Rameshs life time earnings?

400,000 400,000(1.12) 400,000(1.12)14


46
1 2 15

15
1.12

9
1
1.08
= 400,000
0.08 0.12

= Rs.7,254,962

10
Chapter 8

VALUATION OF BONDS AND STOCKS

1. 5 11 100
P = +
t=1 (1.15) (1.15)5

= Rs.11 x PVIFA(15%, 5 years) + Rs.100 x PVIF (15%, 5 years)


= Rs.11 x 3.352 + Rs.100 x 0.497
= Rs.86.7

2.(i) When the discount rate is 14%


7 12 100
P = +
t
t=1 (1.14) (1.14)7

= Rs.12 x PVIFA (14%, 7 years) + Rs.100 x PVIF (14%, 7 years)


= Rs.12 x 4.288 + Rs.100 x 0.4
= Rs.91.46

(ii) When the discount rate is 12%


7 12 100
P = + = Rs.100
t 7
t=1 (1.12) (1.12)

Note that when the discount rate and the coupon rate are the same the value is equal to
par value.

3. The yield to maturity is the value of r that satisfies the following equality.
7 120 1,000
Rs.750 = + = Rs.100
t 7
t=1 (1+r) (1+r)

Try r = 18%. The right hand side (RHS) of the above equation is:
Rs.120 x PVIFA (18%, 7 years) + Rs.1,000 x PVIF (18%, 7 years)
= Rs.120 x 3.812 + Rs.1,000 x 0.314
= Rs.771.44

Try r = 20%. The right hand side (RHS) of the above equation is:
Rs.120 x PVIFA (20%, 7 years) + Rs.1,000 x PVIF (20%, 7 years)
= Rs.120 x 3.605 + Rs.1,000 x 0.279
= Rs.711.60
Thus the value of r at which the RHS becomes equal to Rs.750 lies between 18% and 20%.

11
Using linear interpolation in this range, we get

771.44 750.00
Yield to maturity = 18% + 771.44 711.60 x 2%

= 18.7%
4.
10 14 100
80 = +
t=1 (1+r) t (1+r)10

Try r = 18%. The RHS of the above equation is

Rs.14 x PVIFA (18%, 10 years) + Rs.100 x PVIF (18%, 10 years)


= Rs.14 x 4.494 + Rs.100 x 0.191 = Rs.82

Try r = 20%. The RHS of the above equation is


Rs.14 x PVIFA(20%, 10 years) + Rs.100 x PVIF (20%, 10 years)
= Rs.14 x 4.193 + Rs.100 x 0.162
= Rs.74.9

Using interpolation in the range 18% and 20% we get:

82 - 80
Yield to maturity = 18% + ----------- x 2%
82 74.9

= 18.56%

5.
12 6 100
P = +
t
t=1 (1.08) (1.08)12

= Rs.6 x PVIFA (8%, 12 years) + Rs.100 x PVIF (8%, 12 years)


= Rs.6 x 7.536 + Rs.100 x 0.397
= Rs.84.92

6. The post-tax interest and maturity value are calculated below:

Bond A Bond B

12
* Post-tax interest (C ) 12(1 0.3) 10 (1 0.3)
=Rs.8.4 =Rs.7

* Post-tax maturity value (M) 100 - 100 -


[ (100-70)x 0.1] [ (100 60)x 0.1]
=Rs.97 =Rs.96

The post-tax YTM, using the approximate YTM formula is calculated below

8.4 + (97-70)/10
Bond A : Post-tax YTM = --------------------
0.6 x 70 + 0.4 x 97

= 13.73%

7 + (96 60)/6
Bond B : Post-tax YTM = ----------------------
0.6x 60 + 0.4 x 96

= 17. 47%

7.
14 6 100
P = +
t=1 (1.08) t (1.08)14

= Rs.6 x PVIFA(8%, 14) + Rs.100 x PVIF (8%, 14)


= Rs.6 x 8.244 + Rs.100 x 0.341
= Rs.83.56

8. Do = Rs.2.00, g = 0.06, r = 0.12

Po = D1 / (r g) = Do (1 + g) / (r g)

= Rs.2.00 (1.06) / (0.12 - 0.06)


= Rs.35.33

Since the growth rate of 6% applies to dividends as well as market price, the market
price at the end of the 2nd year will be:

P2 = Po x (1 + g)2 = Rs.35.33 (1.06)2


= Rs.39.70

13
9. Po = D1 / (r g) = Do (1 + g) / (r g)
= Rs.12.00 (1.10) / (0.15 0.10) = Rs.264

10. Po = D1 / (r g)

Rs.32 = Rs.2 / 0.12 g


g = 0.0575 or 5.75%

11. Po = D1/ (r g) = Do(1+g) / (r g)


Do = Rs.1.50, g = -0.04, Po = Rs.8
So
8 = 1.50 (1- .04) / (r-(-.04)) = 1.44 / (r + .04)

Hence r = 0.14 or 14 per cent

12. The market price per share of Commonwealth Corporation will be the sum of three
components:

A: Present value of the dividend stream for the first 4 years


B: Present value of the dividend stream for the next 4 years
C: Present value of the market price expected at the end of 8 years.

A= 1.50 (1.12) / (1.14) + 1.50 (1.12)2 / (1.14)2 + 1.50(1.12)3 / (1.14)3 +


+ 1.50 (1.12)4 / (1.14)4

= 1.68/(1.14) + 1.88 / (1.14)2 + 2.11 / (1.14)3 + 2.36 / (1.14)4


= Rs.5.74

B= 2.36(1.08) / (1.14)5 + 2.36 (1.08)2 / (1.14)6 + 2.36 (1.08)3 / (1.14)7 +


+ 2.36 (1.08)4 / (1.14)8

= 2.55 / (1.14)5 + 2.75 / (1.14)6 + 2.97 / (1.14)7 + 3.21 / (1.14)8


= Rs.4.89

C = P8 / (1.14)8

P8 = D9 / (r g) = 3.21 (1.05)/ (0.14 0.05) = Rs.37.45


So

C = Rs.37.45 / (1.14)8 = Rs.13.14

Thus,
Po = A + B + C = 5.74 + 4.89 + 13.14
= Rs.23.77

14
13. The intrinsic value of the equity share will be the sum of three components:

A: Present value of the dividend stream for the first 5 years when the
growth rate expected is 15%.

B: Present value of the dividend stream for the next 5 years when the
growth rate is expected to be 10%.

C: Present value of the market price expected at the end of 10 years.

2.00 (1.15) 2.00 (1.15)2 2.00 (1.15)3 2.00(1.15)4 2.00 (1.15)5


A= ------------- + ------------- +-------------- + ------------- + -------------
(1.12) (1.12)2 (1.1.2)3 (1.1.2)4 (1.12)5

= 2.30 / (1.12) + 2.65 / (1.12)2 + 3.04 / (1.12)3 + 3.50 / (1.12)4 + 4.02/(1.12)5


= Rs.10.84

4.02(1.10) 4.02 (1.10)2 4.02(1.10)3 4.02(1.10)4 4.02 (1.10)5


B= ------------ + ---------------- + ------------- + --------------- + ---------------
(1.12)6 (1.12)7 (1.12)8 (1..12)9 (1.12)10
4.42 4.86 5.35 5.89 6.48
= --------- + -------------- + --------------- + ------------- + -------------
(1.12)6 (1.12)7 (1.12)8 (1.1.2)9 (1.12)10

= Rs.10.81

D11 1 6.48 (1.05)


C= -------- x --------------- = ------------------- x 1/(1.12)10
rg (1 +r)10 0.12 0.05

= Rs.97.20

The intrinsic value of the share = A + B + C


= 10.84 + 10.81 + 97.20 = Rs.118.85

14. Terminal value of the interest proceeds


= 140 x FVIFA (16%,4)
= 140 x 5.066
= 709.24

Redemption value = 1,000

Terminal value of the proceeds from the bond = 1709.24

15
Define r as the yield to maturity. The value of r can be obtained from the equation

900 (1 + r)4 = 1709.24


r = 0.1739 or 17.39%

15. Intrinsic value of the equity share (using the 2-stage growth model)

(1.18)6
2.36 x 1 - ----------- 2.36 x (1.18)5 x (1.12)
(1.16)6
= --------------------------------- + -----------------------------------
0.16 0.18 (0.16 0.12) x (1.16)6
- 0.10801
= 2.36 x ----------- + 62.05
- 0.02

= Rs.74.80

16. Intrinsic value of the equity share (using the H model)

4.00 (1.20) 4.00 x 4 x (0.10)


= -------------- + ---------------------
0.18 0.10 0.18 0.10

= 60 + 20
= Rs.80

16
Chapter 9
RISK AND RETURN

1 (a) Expected price per share a year hence will be:

= 0.4 x Rs.10 + 0.4 x Rs.11 + 0.2 x Rs.12 = Rs.10.80

(b) Probability distribution of the rate of return is

Rate of return (Ri) 10% 20% 30%

Probability (pi) 0.4 0.4 0.2

Note that the rate of return is defined as:

Dividend + Terminal price


-------------------------------- - 1
Initial price

(c ) The standard deviation of rate of return is : = pi (Ri R)2

The of the rate of return on MVMs stock is calculated below:


---------------------------------------------------------------------------------------------------
Ri pi pI ri (Ri-R) (R i- R)2 pi (Ri-R)2
---------------------------------------------------------------------------------------------------
10 0.4 4 -8 64 25.6
20 0.4 8 2 4 1.6
30 0.2 6 12 144 28.8
---------------------------------------------------------------------------------------------------
R = pi Ri pi (Ri-R)2
= 56
= 56 = 7.48%

2 (a) For Rs.1,000, 20 shares of Alphas stock can be acquired. The probability distribution of the
return on 20 shares is

Economic Condition Return (Rs) Probability


High Growth 20 x 55 = 1,100 0.3
Low Growth 20 x 50 = 1,000 0.3
Stagnation 20 x 60 = 1,200 0.2
Recession 20 x 70 = 1,400 0.2

Expected return = (1,100 x 0.3) + (1,000 x 0.3) + (1,200 x 0.2) + (1,400 x 0.2)

17
= 330 + 300 + 240 + 280
= Rs.1,150

Standard deviation of the return = [(1,100 1,150)2 x 0.3 + (1,000 1,150)2 x

0.3 + (1,200 1,150)2 x 0.2 + (1,400 1,150)2 x 0.2]1/2


= Rs.143.18

(b) For Rs.1,000, 20 shares of Betas stock can be acquired. The probability distribution of the
return on 20 shares is:

Economic condition Return (Rs) Probability

High growth 20 x 75 = 1,500 0.3


Low growth 20 x 65 = 1,300 0.3
Stagnation 20 x 50 = 1,000 0.2
Recession 20 x 40 = 800 0.2

Expected return = (1,500 x 0.3) + (1,300 x 0.3) + (1,000 x 0.2) + (800 x 0.2)
= Rs.1,200

Standard deviation of the return = [(1,500 1,200)2 x .3 + (1,300 1,200)2 x .3


+ (1,000 1,200)2 x .2 + (800 1,200)2 x .2]1/2 = Rs.264.58

(c ) For Rs.500, 10 shares of Alphas stock can be acquired; likewise for Rs.500, 10
shares of Betas stock can be acquired. The probability distribution of this option is:
Return (Rs) Probability
(10 x 55) + (10 x 75) = 1,300 0.3
(10 x 50) + (10 x 65) = 1,150 0.3
(10 x 60) + (10 x 50) = 1,100 0.2
(10 x 70) + (10 x 40) = 1,100 0.2

Expected return = (1,300 x 0.3) + (1,150 x 0.3) + (1,100 x 0.2) +


(1,100 x 0.2)
= Rs.1,175
Standard deviation = [(1,300 1,175)2 x 0.3 + (1,150 1,175)2 x 0.3 +

(1,100 1,175)2 x 0.2 + (1,100 1,175)2 x 0.2 ]1/2


= Rs.84.41
d. For Rs.700, 14 shares of Alphas stock can be acquired; likewise for Rs.300, 6
shares of Betas stock can be acquired. The probability distribution of this
option is:

18
Return (Rs) Probability

(14 x 55) + (6 x 75) = 1,220 0.3


(14 x 50) + (6 x 65) = 1,090 0.3
(14 x 60) + (6 x 50) = 1,140 0.2
(14 x 70) + (6 x 40) = 1,220 0.2

Expected return = (1,220 x 0.3) + (1,090 x 0.3) + (1,140 x 0.2) + (1,220 x 0.2)
= Rs.1,165

Standard deviation = [(1,220 1,165)2 x 0.3 + (1,090 1,165)2 x 0.3 +


(1,140 1,165)2 x 0.2 + (1,220 1,165)2 x 0.2]1/2
= Rs.57.66

The expected return to standard deviation of various options are as follows :


Expected return Standard deviation Expected / Standard
Option (Rs) (Rs) return deviation
a 1,150 143 8.04
b 1,200 265 4.53
c 1,175 84 13.99
d 1,165 58 20.09

Option `d is the most preferred option because it has the highest return to risk ratio.

3. Expected rates of returns on equity stock A, B, C and D can be computed as follows:

A: 0.10 + 0.12 + (-0.08) + 0.15 + (-0.02) + 0.20 = 0.0783 = 7.83%


6

B: 0.08 + 0.04 + 0.15 +.12 + 0.10 + 0.06 = 0.0917 = 9.17%


6

C: 0.07 + 0.08 + 0.12 + 0.09 + 0.06 + 0.12 = 0.0900 = 9.00%


6

D: 0.09 + 0.09 + 0.11 + 0.04 + 0.08 + 0.16 = 0.095 = 9.50%


6

(a) Return on portfolio consisting of stock A = 7.83%

(b) Return on portfolio consisting of stock A and B in equal


proportions = 0.5 (0.0783) + 0.5 (0.0917)
= 0.085 = 8.5%

19
(c ) Return on portfolio consisting of stocks A, B and C in equal
proportions = 1/3(0.0783 ) + 1/3(0.0917) + 1/3 (0.090)
= 0.0867 = 8.67%

(d) Return on portfolio consisting of stocks A, B, C and D in equal


proportions = 0.25(0.0783) + 0.25(0.0917) + 0.25(0.0900) +
0.25(0.095)
= 0.08875 = 8.88%

4. Define RA and RM as the returns on the equity stock of Auto Electricals Limited a and Market
portfolio respectively. The calculations relevant for calculating the beta of the stock are
shown below:

Year RA RM RA-RA RM-RM (RA-RA) (RM-RM) RA-RA/RM-RM


1 15 12 -0.09 -3.18 0.01 10.11 0.29
2 -6 1 -21.09 -14.18 444.79 201.07 299.06
3 18 14 2.91 -1.18 8.47 1.39 -3.43
4 30 24 14.91 8.82 222.31 77.79 131.51
5 12 16 0-3.09 0.82 9.55 0.67 -2.53
6 25 30 9.91 14.82 98.21 219.63 146.87
7 2 -3 -13.09 -18.18 171.35 330.51 237.98
8 20 24 4.91 8.82 24.11 77.79 43.31
9 18 15 2.91 -0.18 8.47 0.03 -0.52
10 24 22 8.91 6.82 79.39 46.51 60.77
11 8. 12 -7.09 -3.18 50.27 10.11 22.55

RA = 15.09 RM = 15.18

(RA RA)2 = 1116.93 (RM RM) 2 = 975.61 (RA RA) (RM RM) = 935.86

Beta of the equity stock of Auto Electricals


(RA RA) (RM RM)

(RM RM) 2

= 935.86 = 0.96
975.61

Alpha = R A A R M

= 15.09 (0.96 x 15.18)= 0.52

20
Equation of the characteristic line is

RA = 0.52 + 0.96 RM

5. The required rate of return on stock A is:

RA = RF + A (RM RF)
= 0.10 + 1.5 (0.15 0.10)
= 0.175

Intrinsic value of share = D1 / (r- g) = Do (1+g) / ( r g)

Given Do = Rs.2.00, g = 0.08, r = 0.175


2.00 (1.08)
Intrinsic value per share of stock A =
0.175 0.08

= Rs.22.74

6. The SML equation is RA = RF + A (RM RF)

Given RA = 15%. RF = 8%, RM = 12%, we have

0.15 = .08 + A (0.12 0.08)

0.07
i.e.A = = 1.75
0.04

Beta of stock A = 1.75

7. The SML equation is: RX = RF + X (RM RF)

We are given 0.15 = 0.09 + 1.5 (RM 0.09) i.e., 1.5 RM = 0.195
or RM = 0.13%

Therefore return on market portfolio = 13%

8. RM = 12% X = 2.0 RX =18% g = 5% Po = Rs.30

Po = D1 / (r - g)

Rs.30 = D1 / (0.18 - .05)

21
So D1 = Rs.39 and Do = D1 / (1+g) = 3.9 /(1.05) = Rs.3.71

Rx = Rf + x (RM Rf)

0.18 = Rf + 2.0 (0.12 Rf)

So Rf = 0.06 or 6%.

Original Revised

Rf 6% 8%
RM Rf 6% 4%
g 5% 4%
x 2.0 1.8

Revised Rx = 8% + 1.8 (4%) = 15.2%

Price per share of stock X, given the above changes is

3.71 (1.04)
= Rs.34.45
0.152 0.04

Chapter 10
OPTIONS AND THEIR VALUATION

1. S = 100 u = 1.5 d = 0.8

22
E = 105 r = 0.12 R = 1.12

The values of (hedge ratio) and B (amount borrowed) can be obtained as follows:

Cu Cd
=
(u d) S

Cu = Max (150 105, 0) = 45

Cd = Max (80 105, 0) = 0

45 0 45 9
= = = = 0.6429
0.7 x 100 70 14

u.Cd d.Cu
B =
(u-d) R

(1.5 x 0) (0.8 x 45)


=
0.7 x 1.12

-36
= = - 45.92
0.784

C = S+B
= 0.6429 x 100 45.92
= Rs.18.37

Value of the call option = Rs.18.37

2. S = 40 u=? d = 0.8
R = 1.10 E = 45 C=8

We will assume that the current market price of the call is equal to the pair value of the call
as per the Binomial model.

Given the above data

Cd = Max (32 45, 0) = 0

23
Cu Cd R
= x
B u Cd d Cu S

Cu 0 1.10
= x
B -0.8Cu 40

= (-) 0.034375

= - 0.34375 B (1)
C = S+B
8 = x 40 + B (2)

Substituting (1) in (2) we get

8 = (-0.034365 x 40) B + B
8 = -0.375 B
or B = - 21.33

= - 0.034375 (-21.33) = 0.7332

The portfolio consists of 0.7332 of a share plus a borrowing of Rs.21.33 (entailing a


repayment of Rs.21.33 (1.10) = Rs.23.46 after one year). It follows that when u occurs either u x 40
x 0.7332 23.46 = u x 40 45
-10.672 u = -21.54
u = 2.02

or

u x 40 x 0.7332 23.46 = 0
u = 0.8

Since u > d, it follows that u = 2.02.


Put differently the stock price is expected to rise by 1.02 x 100 = 102%.

3. Using the standard notations of the Black-Scholes model we get the following results:
ln (S/E) + rt + 2 t/2
d1 =
t

24
= ln (120 / 110) + 0.14 + 0.42/2
0.4

= 0.08701 + 0.14 + 0.08


0.4

= 0.7675

d2 = d1 - t
= 0.7675 0.4
= 0.3675

N(d1) = N (0.7675) ~ N (0.77) = 0.80785


N (d2) = N (0.3675) ~ N (0.37) = 0.64431

C = So N(d1) E. e-rt. N(d2)


= 120 x 0.80785 110 x e-0.14 x 0.64431
= (120 x 0.80785) (110 x 0.86936 x 0.64431)
= 35.33

Value of the call as per the Black and Scholes model is Rs.35.33.

4. t = 0.2 x 1 = 0.2

Ratio of the stock price to the present value of the exercise price
80
= -------------------------
82 x PVIF (15.03,1)

80
= ----------------------
82 x 0.8693
= 1.122

From table A6 we find the percentage relationship between the value of the call option and
stock price to be 14.1 per cent. Hence the value of the call option is
0.141 x 80 = Rs.11,28.

5. Value of put option


= Value of the call option
+ Present value of the exercise price
- Stock price (A)

25
The value of the call option gives an exercise price of Rs.85 can be obtained as follows:

t = 0.2 1 = 0.2

Ratio of the stock price to the present value of the exercise price

80
= ---------------------
85 x PVIF (15.03,1)

= 80 / 73.89 = 1.083

From Table A.6, we find the percentage relationship between the value of the call option and
the stock price to be 11.9%

Hence the value of the call option = 0.119 x 80 = Rs.9.52

Plugging in this value and the other relevant values in (A), we get

Value of put option = 9.52 + 85 x (1.1503)-1 80

= Rs.3.41

6. So = Vo N(d1) B1 e rt N (d2)

= 6000 N (d1) 5000 e 0.1 N(d2)

ln (6000 / 5000) + (0.1 x 1) + (0.18/2)


d1 = ----------------------------------------------
0.18 x 1

ln (1.2) + 0.19
=
0.4243

= 0.8775 = 0.88

N(d1) = N (0.88) = 0.81057


d2 = d1 - t
= 0.8775 - 0.18
= 0.4532 = 0.45

26
N (d2) = N (0.45) = 0.67364
So = 6000 x 0.81057 (5000 x 0.9048 x 0.67364)
= 1816

B0 = V0 S0
= 60000 1816
= 4184

Chapter 11
TECHNIQUES OF CAPITAL BUDGETING

1.(a) NPV of the project at a discount rate of 14%.

= - 1,000,000 + 100,000 + 200,000


---------- ------------
(1.14) (1.14)2

+ 300,000 + 600,000 + 300,000


----------- ---------- ----------
3 4
(1.14) (1.14) (1.14)5

27
= - 44837

(b) NPV of the project at time varying discount rates

= - 1,000,000

+ 100,000

(1.12)

+ 200,000

(1.12) (1.13)

+ 300,000

(1.12) (1.13) (1.14)

+ 600,000

(1.12) (1.13) (1.14) (1.15)

+ 300,000

(1.12) (1.13) (1.14)(1.15)(1.16)

= - 1,000,000 + 89286 + 158028 + 207931 + 361620 + 155871


= - 27264

2. Investment A

a) Payback period = 5 years


b) NPV = 40000 x PVIFA (12,10) 200 000
= 26000
c) IRR (r ) can be obtained by solving the equation:
40000 x PVIFA (r, 10) = 200000
i.e., PVIFA (r, 10) = 5.000

From the PVIFA tables we find that


PVIFA (15,10) = 5.019
PVIFA (16,10) = 4.883

28
Linear interporation in this range yields
r = 15 + 1 x (0.019 / 0.136)
= 15.14%

d) BCR = Benefit Cost Ratio


= PVB / I
= 226,000 / 200,000 = 1.13
Investment B

a) Payback period = 9 years

b) NP V = 40,000 x PVIFA (12,5)


+ 30,000 x PVIFA (12,2) x PVIF (12,5)
+ 20,000 x PVIFA (12,3) x PVIF (12,7)
- 300,000

= (40,000 x 3.605) + (30,000 x 1.690 x 0.567)


+ (20,000 x 2.402 x 0.452) 300,000
= - 105339

c) IRR (r ) can be obtained by solving the equation


40,000 x PVIFA (r, 5) + 30,000 x PVIFA (r, 2) x PVIF (r,5) +
20,000 x PVIFA (r, 3) x PVIF (r, 7) = 300,000

Through the process of trial and error we find that


r = 1.37%

d) BCR = PVB / I
= 194,661 / 300,000 = 0.65

Investment C

a) Payback period lies between 2 years and 3 years. Linear interpolation in this
range provides an approximate payback period of 2.88 years.

b) NPV = 80.000 x PVIF (12,1) + 60,000 x PVIF (12,2)


+ 80,000 x PVIF (12,3) + 60,000 x PVIF (12,4)

+ 80,000 x PVIF (12,5) + 60,000 x PVIF (12,6)


+ 40,000 x PVIFA (12,4) x PVIF (12.6)
- 210,000
= 111,371

29
c) IRR (r) is obtained by solving the equation
80,000 x PVIF (r,1) + 60,000 x PVIF (r,2) + 80,000 x PVIF (r,3)
+ 60,000 x PVIF (r,4) + 80,000 x PVIF (r,5) + 60,000 x PVIF (r,6)
+ 40000 x PVIFA (r,4) x PVIF (r,6) = 210000

Through the process of trial and error we get


r = 29.29%

d) BCR = PVB / I = 321,371 / 210,000 = 1.53

Investment D

a) Payback period lies between 8 years and 9 years. A linear interpolation in this
range provides an approximate payback period of 8.5 years.

8 + (1 x 100,000 / 200,000)

b) NPV = 200,000 x PVIF (12,1)


+ 20,000 x PVIF (12,2) + 200,000 x PVIF (12,9)
+ 50,000 x PVIF (12,10)
- 320,000
= - 37,160

c) IRR (r ) can be obtained by solving the equation


200,000 x PVIF (r,1) + 200,000 x PVIF (r,2)
+ 200,000 x PVIF (r,9) + 50,000 x PVIF (r,10)
= 320000

Through the process of trial and error we get r = 8.45%

d) BCR = PVB / I = 282,840 / 320,000 = 0.88

Comparative Table

Investment A B C D

a) Payback period
(in years) 5 9 2.88 8.5

b) NPV @ 12% pa 26000 -105339 111371-37160

c) IRR (%) 15.14 1.37 29.29 8.45

d) BCR 1.13 0.65 1.53 0.88

30
Among the four alternative investments, the investment to be chosen is C
because it has the Lowest payback period
Highest NPV
Highest IRR
Highest BCR
3. IRR (r) can be calculated by solving the following equations for the value of r.
60000 x PVIFA (r,7) = 300,000

i.e., PVIFA (r,7) = 5.000

Through a process of trial and error it can be verified that r = 9.20% pa.

4. The IRR (r) for the given cashflow stream can be obtained by solving the following equation
for the value of r.

-3000 + 9000 / (1+r) 3000 / (1+r) = 0

Simplifying the above equation we get

r = 1.61, -0.61; (or) 161%, (-)61%

NOTE: Given two changes in the signs of cashflow, we get two values for the
IRR of the cashflow stream. In such cases, the IRR rule breaks down.

5. Define NCF as the minimum constant annual net cashflow that justifies the purchase of the
given equipment. The value of NCF can be obtained from the equation

NCF x PVIFA (10,8) = 500000


NCF = 500000 / 5.335
= 93271

6. Define I as the initial investment that is justified in relation to a net annual cash
inflow of 25000 for 10 years at a discount rate of 12% per annum. The value
of I can be obtained from the following equation

25000 x PVIFA (12,10) = I


i.e., I = 141256

7. PV of benefits (PVB) = 25000 x PVIF (15,1)


+ 40000 x PVIF (15,2)
+ 50000 x PVIF (15,3)
+ 40000 x PVIF (15,4)
+ 30000 x PVIF (15,5)

31
= 122646 (A)

Investment = 100,000 (B)

Benefit cost ratio = 1.23 [= (A) / (B)]

8. The NPVs of the three projects are as follows:

Project
P Q R

Discount rate

0% 400 500 600


5% 223 251 312

10% 69 40 70
15% - 66 - 142 - 135

25% - 291 - 435 - 461


30% - 386 - 555 - 591

9. NPV profiles for Projects P and Q for selected discount rates are as follows:
(a)
Project
P Q
Discount rate (%)
0 2950 500
5 1876 208
10 1075 - 28
15 471 - 222
20 11 - 382
b) (i) The IRR (r ) of project P can be obtained by solving the following
equation for `r.

-1000 -1200 x PVIF (r,1) 600 x PVIF (r,2) 250 x PVIF (r,3)
+ 2000 x PVIF (r,4) + 4000 x PVIF (r,5) = 0

Through a process of trial and error we find that r = 20.13%

(ii) The IRR (r') of project Q can be obtained by solving the following equation for r'

-1600 + 200 x PVIF (r',1) + 400 x PVIF (r',2) + 600 x PVIF (r',3)
+ 800 x PVIF (r',4) + 100 x PVIF (r',5) = 0

32
Through a process of trial and error we find that r' = 9.34%.

c) From (a) we find that at a cost of capital of 10%

NPV (P) = 1075


NPV (Q) = - 28

Given that NPV (P) . NPV (Q); and NPV (P) > 0, I would choose project P.

From (a) we find that at a cost of capital of 20%

NPV (P) = 11

NPV (Q) = - 382

Again NPV (P) > NPV (Q); and NPV (P) > 0. I would choose project P.

d) Project P

PV of investment-related costs

= 1000 x PVIF (12,0)


+ 1200 x PVIF (12,1) + 600 x PVIF (12,2)
+ 250 x PVIF (12,3)
= 2728
TV of cash inflows = 2000 x (1.12) + 4000 = 6240
The MIRR of the project P is given by the equation:
2728 = 6240 x PVIF (MIRR,5)
(1 + MIRR)5 = 2.2874
MIRR = 18%

(c) Project Q

PV of investment-related costs = 1600

TV of cash inflows @ 15% p.a. = 2772

The MIRR of project Q is given by the equation:

16000 (1 + MIRR)5 = 2772

MIRR = 11.62%

33
10
(a) Project A

NPV at a cost of capital of 12%


= - 100 + 25 x PVIFA (12,6)
= Rs.2.79 million

IRR (r ) can be obtained by solving the following equation for r.


25 x PVIFA (r,6) = 100
i.e., r = 12,98%

Project B

NPV at a cost of capital of 12%


= - 50 + 13 x PVIFA (12,6)
= Rs.3.45 million

IRR (r') can be obtained by solving the equation


13 x PVIFA (r',6) = 50
i.e., r' = 14.40% [determined through a process of trial and error]

(b) Difference in capital outlays between projects A and B is Rs.50 million


Difference in net annual cash flow between projects A and B is Rs.12 million.
NPV of the differential project at 12%
= -50 + 12 x PVIFA (12,6)
= Rs.3.15 million

IRR (r'') of the differential project can be obtained from the equation
12 x PVIFA (r'', 6) = 50
i.e., r'' = 11.53%

11
(a) Project M
The pay back period of the project lies between 2 and 3 years. Interpolating in
this range we get an approximate pay back period of 2.63 years/

Project N
The pay back period lies between 1 and 2 years. Interpolating in this range we
get an approximate pay back period of 1.55 years.

(b) Project M

34
Cost of capital = 12% p.a
PV of cash flows up to the end of year 2 = 24.97
PV of cash flows up to the end of year 3 = 47.75
PV of cash flows up to the end of year 4 = 71.26

Discounted pay back period (DPB) lies between 3 and 4 years. Interpolating in this range we
get an approximate DPB of 3.1 years.

Project N
Cost of capital = 12% per annum
PV of cash flows up to the end of year 1 = 33.93
PV of cash flows up to the end of year 2 = 51.47

DPB lies between 1 and 2 years. Interpolating in this range we get an approximate
DPB of 1.92 years.

(c ) Project M
Cost of capital = 12% per annum
NPV = - 50 + 11 x PVIFA (12,1)
+ 19 x PVIF (12,2) + 32 x PVIF (12,3)
+ 37 x PVIF (12,4)
= Rs.21.26 million

Project N
Cost of capital = 12% per annum
NPV = Rs.20.63 million
Since the two projects are independent and the NPV of each project is (+) ve,
both the projects can be accepted. This assumes that there is no capital constraint.
(d) Project M
Cost of capital = 10% per annum
NPV = Rs.25.02 million

Project N
Cost of capital = 10% per annum
NPV = Rs.23.08 million

Since the two projects are mutually exclusive, we need to choose the project with the higher
NPV i.e., choose project M.
NOTE: The MIRR can also be used as a criterion of merit for choosing between the two
projects because their initial outlays are equal.
(e) Project M
Cost of capital = 15% per annum
NPV = 16.13 million

35
Project N
Cost of capital: 15% per annum
NPV = Rs.17.23 million
Again the two projects are mutually exclusive. So we choose the project with the
higher NPV, i.e., choose project N.

(f) Project M
Terminal value of the cash inflows: 114.47
MIRR of the project is given by the equation
50 (1 + MIRR)4 = 114.47
i.e., MIRR = 23.01%

Project N
Terminal value of the cash inflows: 115.41
MIRR of the project is given by the equation
50 ( 1+ MIRR)4 = 115.41
i.e., MIRR = 23.26%

36
Chapter 12
ESTIMATION OF PROJECT CASH FLOWS
1.
(a) Project Cash Flows (Rs. in million)

Year 0 1 2 3 4 5 6 7

1. Plant & machinery (150)

2. Working capital (50)

3. Revenues 250 250 250 250 250 250 250

4. Costs (excluding de-


preciation & interest) 100 100 100 100 100 100 100

5. Depreciation 37.5 28.13 21.09 15.82 11.87 8.90 6.67

6. Profit before tax 112.5 121.87 128.91 134.18 138.13 141.1143.33

7. Tax 33.75 36.56 38.67 40.25 41.44 42.33 43.0

8. Profit after tax 78.75 85.31 90.24 93.93 96.69 98.77100.33

9. Net salvage value of


plant & machinery 48

10. Recovery of working 50


capital

11. Initial outlay (=1+2) (200)

12. Operating CF (= 8 + 5) 116.25 113.44 111.33 109.75 108.56 107.6 107.00

13. Terminal CF ( = 9 +10) 98

14. NCF (200) 116.25 113.44 111.33 109.75 108.56 107.67 205

(c) IRR (r) of the project can be obtained by solving the following equation for r
-200 + 116.25 x PVIF (r,1) + 113.44 x PVIF (r,2)

+ 111.33 x PVIF (r,3) + 109.75 x PVIF (r,4) + 108.56 x PVIF (r,5)


+107.67 x PVIF (r,6) + 205 x PVIF (r,7) = 0

37
Through a process of trial and error, we get r = 55.17%. The IRR of the project is 55.17%.

2. Post-tax Incremental Cash Flows (Rs. in million)

Year 0 1 2 3 4 5 6 7

1. Capital equipment (120)


2. Level of working capital 20 30 40 50 40 30 20
(ending)
3. Revenues 80 120 160 200 160 120 80
4. Raw material cost 24 36 48 60 48 36 24
5. Variable mfg cost. 8 12 16 20 16 12 8
6. Fixed operating & maint. 10 10 10 10 10 10 10
cost
7. Variable selling expenses 8 12 16 20 16 8 12
8. Incremental overheads 4 6 8 10 8 4 6
9. Loss of contribution 10 10 10 10 10 10 10
10.Bad debt loss 4
11. Depreciation 30 22.5 16.88 12.66 9.49 7.12 5.34
12. Profit before tax -14 11.5 35.12 57.34 42.51 26.88 6.66
13. Tax -4.2 3.45 10.54 17.20 12.75 8.06 2.00
14. Profit after tax -9.8 8.05 24.58 40.14 29.76 18.82 4.66
15. Net salvage value of
capital equipments 25
16. Recovery of working 16
capital
17. Initial investment (120)
18. Operating cash flow 20.2 30.55 41.46 52.80 39.25 25.94 14.00
(14 + 10+ 11)
19. Working capital 20 10 10 10 (10) (10) (10)
20. Terminal cash flow 41

21. Net cash flow (140) 10.20 20.55 31.46 62.80 49.25 35.94 55.00
(17+18-19+20)

(b) NPV of the net cash flow stream @ 15% per discount rate

= -140 + 10.20 x PVIF(15,1) + 20.55 x PVIF (15,2)


+ 31.46 x PVIF (15,3) + 62.80 x PVIF (15,4) + 49.25 x PVIF (15,5)
+ 35.94 x PVIF (15,6) + 55 x PVIF (15,7)
= Rs.1.70 million

3.
(a) A. Initial outlay (Time 0)

38
i. Cost of new machine Rs. 3,000,000
ii. Salvage value of old machine 900,000
iii Incremental working capital requirement 500,000
iv. Total net investment (=i ii + iii) 2,600,000

B. Operating cash flow (years 1 through 5)

Year 1 2 3 4 5

i. Post-tax savings in
manufacturing costs 455,000 455,000 455,000 455,000 455,000

ii. Incremental
depreciation 550,000 412,500 309,375 232,031 174,023

iii. Tax shield on


incremental dep. 165,000 123,750 92,813 69,609 52,207
iv. Operating cash
flow ( i + iii) 620,000 578,750 547,813 524,609 507,207

C. Terminal cash flow (year 5)

i. Salvage value of new machine Rs. 1,500,000


ii. Salvage value of old machine 200,000
iii. Recovery of incremental working capital 500,000
iv. Terminal cash flow ( i ii + iii) 1,800,000

D. Net cash flows associated with the replacement project (in Rs)

Year 0 1 2 3 4 5
NCF (2,600,000) 620000 578750 547813 524609 2307207

(b) NPV of the replacement project


= - 2600000 + 620000 x PVIF (14,1)
+ 578750 x PVIF (14,2)
+ 547813 x PVIF (14,3)
+ 524609 x PVIF (14,4)
+ 2307207 x PVIF (14,5)
= Rs.267849

4. Tax shield (savings) on depreciation (in Rs)

39
Depreciation Tax shield PV of tax shield
Year charge (DC) =0.4 x DC @ 15% p.a.

1 25000 10000 8696

2 18750 7500 5671

3 14063 5625 3699

4 10547 4219 2412

5 7910 3164 1573


----------
22051
----------

Present value of the tax savings on account of depreciation = Rs.22051

5. A. Initial outlay (at time 0)


i. Cost of new machine Rs. 400,000
ii. Salvage value of the old machine 90,000
iii. Net investment 310,000

B. Operating cash flow (years 1 through 5)

Year 1 2 3 4 5
i. Depreciation
of old machine 18000 14400 11520 9216 7373

ii. Depreciation
of new machine 100000 75000 56250 42188 31641

iii. Incremental
depreciation
( ii i) 82000 60600 44730 32972 24268

iv. Tax savings on


incremental
depreciation
( 0.35 x (iii)) 28700 21210 15656 11540 8494

v. Operating cash
flow 28700 21210 15656 11540 8494
C. Terminal cash flow (year 5)

40
i. Salvage value of new machine Rs. 25000
ii. Salvage value of old machine 10000
iii. Incremental salvage value of new
machine = Terminal cash flow 15000

D. Net cash flows associated with the replacement proposal.

Year 0 1 2 3 4 5

NCF (310000) 28700 21210 15656 11540 23494

MINICASE
Solution:

a. Cash flows from the point of all investors (which is also called the explicit cost funds point of
view)
Rs.in million

Item 0 1 2 3 4 5

1. Fixed assets (15)


2. Net working
capital (8)
3. Revenues 30 30 30 30 30
4. Costs (other than
depreciation and
interest) 20 20 20 20 20
5. Loss of rental 1 1 1 1 1
6. Depreciation 3.750 2.813 2.109 1.582 1.187
7. Profit before tax 5.250 6.187 6.891 7.418 7.813
8. Tax 1.575 1.856 2.067 2.225 2.344
9. Profit after tax 3.675 4.331 4.824 5.193 5.469
10. Salvage value of
fixed assets 5.000
11. Net recovery of
working capital 8.000

12. Initial outlay (23)


13. Operating cash
inflow 7.425 7.144 6.933 6.775 6.656
14. Terminal cash

41
flow 13.000
15. Net cash flow (23) 7.425 7.144 6.933 6.775 19.656

b. Cash flows form the point of equity investors

Rs.in million

Item 0 1 2 3 4 5

1. Equity funds (10)


2. Revenues 30 30 30 30 30
3. Costs (other than
depreciation and
interest) 20 20 20 20 20
4. Loss of rental 1 1 1 1 1
5. Depreciation 3.75 2.813 2.109 1.582 1.187
6. Interest on working
capital advance 0.70 0.70 0.70 0.70 0.70
7. Interest on term
loans 1.20 1.125 0.825 0.525 0.225
8. Profit before tax 3.35 4.362 5.366 6.193 6.888
9. Tax 1.005 1.309 1.610 1.858 2.066
10. Profit after tax 2.345 3.053 3.756 4.335 4.822
11. Net salvage value
of fixed assets 5.000
12. Net salvage value
of current assets 10.000
13. Repayment of term
term loans 2.000 2.000 2.000 2.000
14. Repayment of bank
advance 5.000
15. Retirement of trade
creditors 2.000
16. Initial investment (10)
17. Operating cash
inflow 6.095 5.866 5.865 5.917 6.009
18. Liquidation and
retirement cash
flows (2.0) (2.0) (2.0) 6.00
19. Net cash flow (10) 6.095 3.866 3.865 3.917 12.009

42
Chapter 13
RISK ANALYSIS IN CAPITAL BUDGETING

1.
(a) NPV of the project = -250 + 50 x PVIFA (13,10)
= Rs.21.31 million

(b) NPVs under alternative scenarios:


(Rs. in million)
Pessimistic Expected Optimistic

Investment 300 250 200


Sales 150 200 275
Variable costs 97.5 120 154
Fixed costs 30 20 15
Depreciation 30 25 20
Pretax profit - 7.5 35 86
Tax @ 28.57% - 2.14 10 24.57
Profit after tax - 5.36 25 61.43
Net cash flow 24.64 50 81.43
Cost of capital 14% 13% 12%

NPV - 171.47 21.31 260.10

Assumptions: (1) The useful life is assumed to be 10 years under all three
scenarios. It is also assumed that the salvage value of the
investment after ten years is zero.

(2) The investment is assumed to be depreciated at 10% per annum; and it


is also assumed that this method and rate of depreciation are
acceptable to the IT (income tax) authorities.

(3) The tax rate has been calculated from the given table i.e. 10 / 35 x 100
= 28.57%.

(4) It is assumed that only loss on this project can be offset against the
taxable profit on other projects of the company; and thus the company
can claim a tax shield on the loss in the same year.

(c) Accounting break even point (under expected scenario)


Fixed costs + depreciation = Rs. 45 million

43
Contribution margin ratio = 60 / 200 = 0.3
Break even level of sales = 45 / 0.3 = Rs.150 million

Financial break even point (under xpected scenario)

i. Annual net cash flow = 0.7143 [ 0.3 x sales 45 ] + 25


= 0.2143 sales 7.14

ii. PV (net cash flows) = [0.2143 sales 7.14 ] x PVIFA (13,10)


= 1.1628 sales 38.74

iii. Initial investment = 200

iv. Financial break even level


of sales = 238.74 / 1.1628 = Rs.205.31 million

2.
(a) Sensitivity of NPV with respect to quantity manufactured and sold:
(in Rs)
Pessimistic Expected Optimistic

Initial investment 30000 30000 30000


Sale revenue 24000 42000 54000
Variable costs 16000 28000 36000
Fixed costs 3000 3000 3000
Depreciation 2000 2000 2000
Profit before tax 3000 9000 13000
Tax 1500 4500 6500
Profit after tax 1500 4500 6500
Net cash flow 3500 6500 8500
NPV at a cost of
capital of 10% p.a
and useful life of
5 years -16732 - 5360 2222

(b) Sensitivity of NPV with respect to variations in unit price.

Pessimistic Expected Optimistic

Initial investment 30000 30000 30000


Sale revenue 28000 42000 70000
Variable costs 28000 28000 28000
Fixed costs 3000 3000 3000

44
Depreciation 2000 2000 2000
Profit before tax -5000 9000 37000
Tax -2500 4500 18500
Profit after tax -2500 4500 18500
Net cash flow - 500 6500 20500
NPV - 31895 (-) 5360 47711

(c) Sensitivity of NPV with respect to variations in unit variable cost.

Pessimistic Expected Optimistic

Initial investment 30000 30000 30000


Sale revenue 42000 42000 42000
Variable costs 56000 28000 21000
Fixed costs 3000 3000 3000
Depreciation 2000 2000 2000
Profit before tax -11000 9000 16000
Tax -5500 4500 8000
Profit after tax -5500 4500 8000
Net cash flow -3500 6500 10000
NPV -43268 - 5360 7908

(d) Accounting break-even point

i. Fixed costs + depreciation = Rs.5000


ii. Contribution margin ratio = 10 / 30 = 0.3333
iii. Break-even level of sales = 5000 / 0.3333
= Rs.15000
Financial break-even point

i. Annual cash flow = 0.5 x (0.3333 Sales 5000) = 2000


ii. PV of annual cash flow = (i) x PVIFA (10,5)
= 0.6318 sales 1896
iii. Initial investment = 30000
iv. Break-even level of sales = 31896 / 0.6318 = Rs.50484

3. Define At as the random variable denoting net cash flow in year t.

A1 = 4 x 0.4 + 5 x 0.5 + 6 x 0.1


= 4.7

A2 = 5 x 0.4 + 6 x 0.4 + 7 x 0.2


= 5.8

45
A3 = 3 x 0.3 + 4 x 0.5 + 5 x 0.2
= 3.9

NPV = 4.7 / 1.1 +5.8 / (1.1)2 + 3.9 / (1.1)3 10


= Rs.2.00 million

12 = 0.41

22 = 0.56

32 = 0.49

12 22 32
2NPV = + +
2 4
(1.1) (1.1) (1.1)6
= 1.00
(NPV) = Rs.1.00 million

4. Expected NPV
4 At
= - 25,000
t=1 (1.08)t

= 12,000/(1.08) + 10,000 / (1.08)2 + 9,000 / (1.08)3


+ 8,000 / (1.08)4 25,000

= [ 12,000 x .926 + 10,000 x .857 + 9,000 x .794 + 8,000 x .735]


- 25,000
= 7,708

Standard deviation of NPV


4 t

t=1 (1.08)t

= 5,000 / (1.08) + 6,000 / (1.08)2 + 5,000 / (1,08)3 + 6,000 / (1.08)4


= 5,000 x .926 + 6,000 x .857 + 5000 x .794 + 6,000 x .735
= 18,152

5. Expected NPV
4 At
= - 10,000 . (1)

46
t=1 (1.06)t
A1 = 2,000 x 0.2 + 3,000 x 0.5 + 4,000 x 0.3
= 3,100

A2 = 3,000 x 0.4 + 4,000 x 0.3 + 5,000 x 0.3


= 3,900

A3 = 4,000 x 0.3 + 5,000 x 0.5 + 6,000 x 0.2


= 4,900

A4 = 2,000 x 0.2 + 3,000 x 0.4 + 4,000 x 0.4


= 3,200

Substituting these values in (1) we get

Expected NPV = NPV

= 3,100 / (1.06)+ 3,900 / 1.06)2 + 4,900 / (1.06)3 + 3,200 / (1,06)4


- 10,000 = Rs.3,044

The variance of NPV is given by the expression

4 2t
2 (NPV) = .. (2)
t=1 (1.06)2t

12 = [(2,000 3,100)2 x 0.2 + (3,000 3,100)2 x 0.5


+ (4,000 3,100)2 x 0.3]
= 490,000

22 = [(3,000 3,900)2 x 0.4 + (4,000 3,900)2 x 0.3


+ (5,000 3900)2 x 0.3]
= 690,000

32 = [(4,000 4,900)2 x 0.3 + (5,000 4,900)2 x 0.5


+ (6,000 4,900)2 x 0.2]
= 490,000

42 = [(2,000 3,200)2 x 0.2 + (3,000 3,200)2 x 0.4


+ (4,000 3200)2 x 0.4]
= 560,000
Substituting these values in (2) we get
490,000 / (1.06)2 + 690,000 / (1.06)4

47
+ 490,000 / (1.06)6 + 560,000 / (1.08)8
[ 490,000 x 0.890 + 690,000 x 0.792
+ 490,000 x 0.705 + 560,000 x 0.627 ]
= 1,679,150
NPV = 1,679,150 = Rs.1,296

NPV NPV 0 - NPV


Prob (NPV < 0) = Prob. <
NPV NPV
0 3044
= Prob Z<
1296

= Prob (Z < -2.35)

The required probability is given by the shaded area in the following normal curve.

P (Z < - 2.35) = 0.5 P (-2.35 < Z < 0)


= 0.5 P (0 < Z < 2.35)
= 0.5 0.4906
= 0.0094

So the probability of NPV being negative is 0.0094

Prob (P1 > 1.2) Prob (PV / I > 1.2)


Prob (NPV / I > 0.2)
Prob. (NPV > 0.2 x 10,000)
Prob (NPV > 2,000)

Prob (NPV >2,000)= Prob (Z > 2,000- 3,044 / 1,296)


Prob (Z > - 0.81)

The required probability is given by the shaded area of the following normal
curve:
P(Z > - 0.81) = 0.5 + P(-0.81 < Z < 0)
= 0.5 + P(0 < Z < 0.81)
= 0.5 + 0.2910
= 0.7910

So the probability of P1 > 1.2 as 0.7910

6. Given values of variables other than Q, P and V, the net present value model of Bidhan
Corporation can be expressed as:

48
[Q(P V) 3,000 2,000] (0.5)+ 2,000 0
5
NPV + - 30,000
t 5
t =1 (1.1) (1.1)

0.5 Q (P V) 500
5
= ------------------------------------ - 30,000
t=1 (1.1)t

= [ 0.5Q (P V) 500] x PVIFA (10,5) 30,000


= [0.5Q (P V) 500] x 3.791 30,000
= 1.8955Q (P V) 31,895.5

Exhibit 1 presents the correspondence between the values of exogenous variables and the two
digit random number. Exhibit 2 shows the results of the simulation.
Exhibit 1
Correspondence between values of exogenous variables and
two digit random numbers

QUANTITY PRICE VARIABLE COST


Two digit Two digit Two digit
Cumulati random Cumulati random Cum random
Valu Pro ve Prob. numbers Valu Pro ve Prob. numbers Value Pro u- numbers
e b e b b lative
Prob.
800 0.1 0.10 00 to 09 20 0.4 0.40 00 to 39 15 0.3 0.30 00 to 29
0 0 0
1,00 0.1 0.20 10 to 19 30 0.4 0.80 40 to 79 20 0.5 0.80 30 to 79
0 0 0 0
1,20 0.2 0.40 20 to 39 40 0.1 0.90 80 to 89 40 0.2 1.00 80 to 99
0 0 0 0
1,40 0.3 0.70 40 to 69 50 0.1 1.00 90 to 99
0 0 0
1,60 0.2 0.90 70 to 89
0 0
1,80 0.1 1.00 90 to 99
0 0

Exhibit 2
Simulation Results

49
QUANTITY (Q) PRICE (P) VARIABLE COST (V) NPV
Ru Rando Corres- Random Corres- Rando Corres- 1.8955 Q(P-V)-
n m ponding Number ponding m pondin 31,895.5
Numb Value value Numbe g value
er r
1 03 800 38 20 17 15 -24,314
2 32 1,200 69 30 24 15 2,224
3 61 1,400 30 20 03 15 -18,627
4 48 1,400 60 30 83 40 -58,433
5 32 1,200 19 20 11 15 -20,523
6 31 1,200 88 40 30 20 13,597
7 22 1,200 78 30 41 20 -9,150
8 46 1,400 11 20 52 20 -31,896
9 57 1,400 20 20 15 15 -18,627
QUANTITY (Q) PRICE (P) VARIABLE COST (V) NPV
Ru Rando Corres- Random Corres- Rando Corres- 1.8955 Q(P-V)-
n m ponding Number ponding m pondin 31,895.5
Numb Value value Numbe g value
er r
10 92 1,800 77 30 38 20 2,224
11 25 1,200 65 30 36 20 -9,150
12 64 1,400 04 20 83 40 -84,970
13 14 1,000 51 30 72 20 -12,941
14 05 800 39 20 81 40 -62,224
15 07 800 90 50 40 20 13,597
16 34 1,200 63 30 67 20 -9,150
17 79 1,600 91 50 99 40 -1,568
18 55 1,400 54 30 64 20 -5,359
19 57 1,400 12 20 19 15 -18,627
20 53 1,400 78 30 22 15 7,910
21 36 1,200 79 30 96 40 -54,642
22 32 1,200 22 20 75 20 -31,896
23 49 1,400 93 50 88 40 -5,359
24 21 1,200 84 40 35 20 13,597
25 08 .800 70 30 27 15 -9,150
26 85 1,600 63 30 69 20 -1,568
27 61 1,400 68 30 16 15 7,910
28 25 1,200 81 40 39 20 13,597
29 51 1,400 76 30 38 20 -5,359
30 32 1,200 47 30 46 20 -9,150
31 52 1,400 61 30 58 20 -5,359
32 76 1,600 18 20 41 20 -31,896

50
33 43 1,400 04 20 49 20 -31,896
34 70 1,600 11 20 59 20 -31,896
35 67 1,400 35 20 26 15 -18,627
36 26 1,200 63 30 22 15 2,224

QUANTITY (Q) PRICE (P) VARIABLE COST (V) NPV


Ru Random Corres Random Corres- Rando Corres- 1.8955 Q(P-V)-
n Number - Number ponding m pondin 31,895.5
pondi value Numbe g value
ng r
Value
37 89 1,600 86 40 59 20 28,761
38 94 1,800 00 20 25 15 -14,836
39 09 .800 15 20 29 15 -24,314
40 44 1,400 84 40 21 15 34,447
41 98 1,800 23 20 79 20 -31,896
42 10 1,000 53 30 77 20 -12,941
43 38 1,200 44 30 31 20 -9,150
44 83 1,600 30 20 10 15 -16,732
45 54 1,400 71 30 52 20 -5,359
46 16 1,000 70 30 19 15 -3,463
47 20 1,200 65 30 87 40 -54,642
48 61 1,400 61 30 70 20 -5,359
49 82 1,600 48 30 97 40 -62,224
50 90 1,800 50 30 43 20 2,224

Expected NPV = NPV


50
= 1/ 50 NPVi
i=1
= 1/50 (-7,20,961)
= 14,419

50
Variance of NPV = 1/50 NPVi NPV)2
i=1

= 1/50 [27,474.047 x 106]


= 549.481 x 106

Standard deviation of NPV = 549.481 x 106

51
= 23,441

7. To carry out a sensitivity analysis, we have to define the range and the most likely values of
the variables in the NPV Model. These values are defined below

Variable Range Most likely value

I Rs.30,000 Rs.30,000 Rs.30,000


k 10% - 10% 10%
F Rs.3,000 Rs.3,000 Rs.3,000
D Rs.2,000 Rs.2,000 Rs.2,000
T 0.5 0.5 0.5
N 55 5
S 00 0
Q Can assume any one of the values - 1,400*
800, 1,000, 1,200, 1,400, 1,600 and 1,800
P Can assume any of the values 20, 30, 30**
40 and 50
V Can assume any one of the values 20*
15,20 and 40
----------------------------------------------------------------------------------------
* The most likely values in the case of Q, P and V are the values that have the
highest probability associated with them

** In the case of price, 20 and 30 have the same probability of occurrence viz 0.4. We
have chosen 30 as the most likely value because the expected value of the
distribution is closer to 30

Sensitivity Analysis with Reference to Q

The relationship between Q and NPV given the most likely values of other
variables is given by

5 [Q (30-20) 3,000 2,000] x 0.5 + 2,000 0


NPV = + - 30,000
t=1 (1.1)t (1.1)5

5 5Q - 500
= - 30,000
t=1 (1.1)t

The net present values for various values of Q are given in the following table:

Q 800 1,000 1,200 1,400 1,600 1,800

52
NPV -16,732 -12,941 -9,150 -5,359 -1,568 2,224

Sensitivity analysis with reference to P

The relationship between P and NPV, given the most likely values of other variables is defined as
follows:

5 [1,400 (P-20) 3,000 2,000] x 0.5 + 2,000 0


NPV = + - 30,0
t 5
t=1 (1.1) (1.1)

5 700 P 14,500
= - 30,000
t=1 (1.1)t

The net present values for various values of P are given below :
P (Rs) 20 30 - 40 50
NPV(Rs) -31,896 -5,359 21,179 47,716

8. NPV -5 0 5 10 15 20
(Rs.in lakhs)
PI 0.9 1.00 1.10 1.20 1.30 1.40

Prob. 0.02 0.03 0.10 0.40 0.30 0.15

6
Expected PI = PI = (PI)j P j
j=1
= 1.24

6
Standard deviation of P1 = (PIj - PI) 2 P j
j=1
= .01156
= .1075
The standard deviation of P1 is .1075 for the given investment with an expected PI of 1.24.
The maximum standard deviation of PI acceptable to the company for an investment with an
expected PI of 1.25 is 0.30.

Since the risk associated with the investment is much less than the maximum risk acceptable
to the company for the given level of expected PI, the company must should accept the
investment.

53
9. The NPVs of the two projects calculated at their risk adjusted discount rates are as follows:
6 3,000
Project A: NPV = - 10,000 = Rs.2,333
t
t=1 (1.12)

5 11,000
Project B: NPV = - 30,000 = Rs.7,763
t
t=1 (1.14)

PI and IRR for the two projects are as follows:

Project A B

PI 1.23 1.26
IRR 20% 24.3%

B is superior to A in terms of NPV, PI, and IRR. Hence the company must choose B.

10. The certainty equivalent co-efficients for the five years are as follows

Year Certainty equivalent coefficient

t = 1 0.06 t

1 1 = 0.94
2 2 = 0.88
3 3 = 0.82
4 = 0.76
5 = 0.70

The present value of the project calculated at the risk-free rate of return is :
5 (1 0.06 t) At

t=1 (1.08)t
7,000 x 0.94 8,000 x 0.88 9,000 x 0.82 10,000 x 0.76 8,000 x 0.70
+ + + +
2 3 4
(1.08) (1.08) (1.08) (1.08) (1.08)5

6,580 7,040 7,380 7,600 5,600


+ + + +
(1.08) (1.08)2 (1.08)3 (1.08)4 (1.08)5

= 27,386

54
Net present value of the Project = (27,386 30,000
= Rs. 2,614

MINICASE

Solution:
1. The expected NPV of the turboprop aircraft

0.65 (5500) + 0.35 (500)


NPV = - 11000 +
(1.12)

0.65 [0.8 (17500) + 0.2 (3000)] + 0.35 [0.4 (17500) + 0.6 (3000)]
+
(1.12)2
= 2369

2. If Southern Airways buys the piston engine aircraft and the demand in year 1 turns out to be
high, a further decision has to be made with respect to capacity expansion. To evaluate the
piston engine aircraft, proceed as follows:

First, calculate the NPV of the two options viz., expand and do not expand at decision
point D2:

0.8 (15000) + 0.2 (1600)


Expand : NPV = - 4400 +
1.12

= 6600

0.8 (6500) + 0.2 (2400)


Do not expand : NPV =
1.12
= 5071

Second, truncate the do not expand option as it is inferior to the expand option. This
means that the NPV at decision point D2 will be 6600

Third, calculate the NPV of the piston engine aircraft option.

0.65 (2500+6600) + 0.35 (800)

55
NPV = 5500 +
1.12

0.35 [0.2 (6500) + 0.8 (2400)]


+
(1.12)2

= 5500 + 5531 + 898 = 929

3. The value of the option to expand in the case of piston engine aircraft
If Southern Airways does not have the option of expanding capacity at the end of year 1, the
NPV of the piston engine aircraft would be:

0.65 (2500) + 0.35 (800)


NPV = 5500 +
1.12

0.65 [0.8 (6500) + 0.2 (2400)] + 0.35 [0.2 (6500) + 0.8 (2400)]
+
(1.12)2

= - 5500 + 1701 + 3842 = 43

Thus the option to expand has a value of 929 43 = 886

4. Value of the option to abandon if the turboprop aircraft can be sold for 8000 at the end of year
1

If the demand in year 1 turns out to be low, the payoffs for the continuation and
abandonment options as of year 1 are as follows.

0.4 (17500) + 0.6 (3000)


Continuation: = 7857
1.12

Abandonment : 8000

Thus it makes sense to sell off the aircraft after year 1, if the demand in year 1 turns out to be
low.

The NPV of the turboprop aircraft with abandonment possibility is

56
0.65 [5500 +{0.8 (17500) + 0.2 (3000)}/ (1.12)] + 0.35 (500 +8000)
NPV = - 11,000 +
(1.12)

12048 + 2975
= - 11,000 + = 2413
1.12

Since the turboprop aircraft without the abandonment option has a value of 2369, the
value of the abandonment option is : 2413 2369 = 44

5. The value of the option to abandon if the piston engine aircraft can be sold for 4400 at the
end of year 1:

If the demand in year 1 turns out to be low, the payoffs for the continuation and
abandonment options as of year 1 are as follows:

0.2 (6500) + 0.8 (2400)


Continuation : = 2875
1.12

Abandonment : 4400

Thus, it makes sense to sell off the aircraft after year 1, if the demand in year 1 turns out to be
low.

The NPV of the piston engine aircraft with abandonment possibility is:

0.65 [2500 + 6600] + 0.35 [800 + 4400]


NPV = - 5500 +
1.12

5915 + 1820
= - 5500 + = 1406
1.12

For the piston engine aircraft the possibility of abandonment increases the NPV
from 929 to 1406. Hence the value of the abandonment option is 477.

57
Chapter 14
THE COST OF CAPITAL

1(a) Define rD as the pre-tax cost of debt. Using the approximate yield formula, rD can be
calculated as follows:

14 + (100 108)/10
rD = ------------------------ x 100 = 12.60%
0.4 x 100 + 0.6x108

(b) After tax cost = 12.60 x (1 0.35) = 8.19%

2. Define rp as the cost of preference capital. Using the approximate yield formula rp can be
calculated as follows:

9 + (100 92)/6
rp = --------------------
0.4 x100 + 0.6x92

= 0.1085 (or) 10.85%

3. WACC = 0.4 x 13% x (1 0.35)


+ 0.6 x 18%
= 14.18%

4. Cost of equity = 10% + 1.2 x 7% = 18.4%


(using SML equation)

Pre-tax cost of debt = 14%

After-tax cost of debt = 14% x (1 0.35) = 9.1%

Debt equity ratio = 2:3

WACC = 2/5 x 9.1% + 3/5 x 18.4%

= 14.68%

5. Given
0.5 x 14% x (1 0.35) + 0.5 x rE = 12%

where rE is the cost of equity capital.

Therefore rE 14.9%

58
Using the SML equation we get

11% + 8% x = 14.9%

where denotes the beta of Azeezs equity.

Solving this equation we get = 0.4875.

6(a) The cost of debt of 12% represents the historical interest rate at the time the debt was
originally issued. But we need to calculate the marginal cost of debt (cost of raising new
debt); and for this purpose we need to calculate the yield to maturity of the debt as on the
balance sheet date. The yield to maturity will not be equal to12% unless the book value of
debt is equal to the market value of debt on the balance sheet date.

(b) The cost of equity has been taken as D1/P0 ( = 6/100) whereas the cost of equity is (D1/P0)
+ g where g represents the expected constant growth rate in dividend per share.

7. The book value and market values of the different sources of finance are
provided in the following table. The book value weights and the market value
weights are provided within parenthesis in the table.
(Rs. in million)
Source Book value Market value
Equity 800 (0.54) 2400 (0.78)
Debentures first series 300 (0.20) 270 (0.09)
Debentures second series 200 (0.13) 204 (0.06)
Bank loan 200 (0.13) 200 (0.07)
Total 1500 (1.00) 3074 (1.00)

8. Required return
based on SML Expected
Project Beta equation (%) return (%)

P 0.6 14.8 13
Q 0.9 17.2 14
R 1.5 22.0 16
S 1.5 22.0 20

Given a hurdle rate of 18% (the firms cost of capital), projects P, Q and R would have been
rejected because the expected returns on these projects are below 18%. Project S would be
accepted because the expected return on this project exceeds 18%.An appropriate basis for

59
accepting or rejecting the projects would be to compare the expected rate of return and the
required rate of return for each project. Based on this comparison, we find that all the four
projects need to be rejected.

9.
(a) Given
rD x (1 0.3) x 4/9 + 20% x 5/9 = 15%
rD = 12.5%,where rD represents the pre-tax cost of debt.

(b) Given
13% x (1 0.3) x 4/9 + rE x 5/9 = 15%
rE = 19.72%, where rE represents the cost of equity.

10. Cost of equity = D1/P0 + g


= 3.00 / 30.00 + 0.05
= 15%
(a) The first chunk of financing will comprise of Rs.5 million of retained earnings costing 15
percent and Rs.25 million of debt costing 14 (1-.3) = 9.8 per cent
The second chunk of financing will comprise of Rs.5 million of additional equity costing
15 per cent and Rs.2.5 million of debt costing 15 (1-.3) = 10.5 per cent

(b) The marginal cost of capital in the first chunk will be :


5/7.5 x 15% + 2.5/7.5 x 9.8% = 13.27%

The marginal cost of capital in the second chunk will be :


5/7.5 x 15% + 2.5/7.5 x 10.5% = 13.50%

Note : We have assumed that


(i) The net realisation per share will be Rs.25, after floatation costs, and
(ii) The planned investment of Rs.15 million is inclusive of floatation costs

11. The cost of equity and retained earnings


rE = D1/PO + g
= 1.50 / 20.00 + 0.07 = 14.5%
The cost of preference capital, using the approximate formula, is :

11 + (100-75)/10
rE = = 15.9%
0.6 x 75 + 0.4 x 100

60
The pre-tax cost of debentures, using the approximate formula, is :

13.5 + (100-80)/6
rD = = 19.1%
0.6x80 + 0.4x100

The post-tax cost of debentures is


19.1 (1-tax rate) = 19.1 (1 0.5)
= 9.6%

The post-tax cost of term loans is


12 (1-tax rate) = 12 (1 0.5)
= 6.0%

The average cost of capital using book value proportions is calculated below :

Source of capital Component Book value Book value Product of


Cost Rs. in million proportion (1) & (3)
(1) (2) (3)
Equity capital 14.5% 100 0.28 4.06
Preference capital 15.9% 10 0.03 0.48
Retained earnings 14.5% 120 0.33 4.79
Debentures 9.6% 50 0.14 1.34
Term loans 6.0% 80 0.22 1.32
360 Average cost11.99%
capital

The average cost of capital using market value proportions is calculated below :

Source of capital Component Market value Market value Product of


cost Rs. in million
(1) (2) (3) (1) & (3)

Equity capital
and retained earnings 14.5% 200 0.62 8.99
Preference capital 15.9% 7.5 0.02 0.32
Debentures 9.6% 40 0.12 1.15
Term loans 6.0% 80 0.24 1.44

327.5 Average cost 11.90%


capital

12

61
(a) WACC = 1/3 x 13% x (1 0.3)
+ 2/3 x 20%
= 16.37%

(b) Weighted average floatation cost


= 1/3 x 3% + 2/3 x 12%
= 9%

(c) NPV of the proposal after taking into account the floatation costs
= 130 x PVIFA (16.37, 8) 500 / (1 - 0.09)
= Rs.8.51 million

MINICASE

Solution:

a. All sources other than non-interest bearing liabilities

b. Pre-tax cost of debt & post-tax cost of debt

10 + (100 112) / 8 8.5


rd = = = 7.93
0.6 x 112 + 0.4 x 100 107.2

rd (1 0.3) = 5.55

c. Post-tax cost of preference


9 + (100 106) / 5 7.8
= = 7.53%
0.6 x 106 + 0.4 x 100 103.6
d. Cost of equity using the DDM

2.80 (1.10)
+ 0.10 = 0.385 + 0.10
80
= 0.1385 = 13.85%

e. Cost of equity using the CAPM

7 + 1.1(7) = 14.70%

f. WACC
0.50 x 14.70 + 0.10 x 7.53 + 0.40 x 5.55

62
= 7.35 + 0.75 + 2.22
= 10.32%

g. Cost of capital for the new business

0.5 [7 + 1.5 (7)] + 0.5 [ 11 (1 0.3)]


8.75 + 3.85
= 12.60%

63
Chapter 15
CAPITAL BUDGETING : EXTENSIONS

1. EAC
(Plastic Emulsion) = 300000 / PVIFA (12,7)
= 300000 / 4.564
= Rs.65732

EAC
(Distemper Painting) = 180000 / PVIFA (12,3)
= 180000 / 2.402
= Rs.74938

Since EAC of plastic emulsion is less than that of distemper painting, it is the preferred
alternative.

2. PV of the net costs associated with the internal transportation system

= 1 500 000 + 300 000 x PVIF (13,1) + 360 000 x PVIF (13,2)
+ 400 000 x PVIF (13,3) + 450 000 x PVIF (13,4)
+ 500 000 x PVIF (13,5) - 300 000 x PVIF (13,5)
= 2709185

EAC of the internal transportation system

= 2709185 / PVIFA (13,5)


= 2709185 / 3.517
= Rs.770 311

3. EAC [ Standard overhaul]

= 500 000 / PVIFA (14,6)


= 500 000 / 3.889
= Rs.128568 (A)

EAC [Less costly overhaul]

= 200 000 / PVIFA (14,2)


= 200 000 / 1.647
= Rs.121433 (B)

Since (B) < (A), the less costly overhaul is preferred alternative.

64
4.
(a) Base case NPV

= -12,000,000 + 3,000,000 x PVIFA (20,6)


= -12,000,000 + 997,8000
= (-) Rs.2,022,000

(b) Issue costs = 6,000,000 / 0.88 - 6,000,000

= Rs.818 182

Adjusted NPV after adjusting for issue costs

= - 2,022,000 818,182
= - Rs.2,840,182

(c) The present value of interest tax shield is calculated below :

Year Debt outstanding at Interest Tax shield Present value of


the beginning tax shield
1 6,000,000 1,080,000 324,000 274,590
2 6,000,000 1,080,000 324,000 232,697
3 5,250,000 945,000 283,000 172,538
4 4,500,000 810,000 243,000 125,339
5 3,750,000 675,000 202,000 88,513
6 3,000,000 540,000 162,000 60,005
7 2,225,000 400,500 120,000 37,715
8 1,500,000 270,000 81,000 21,546
9 750,000 135,000 40,500 9,133

Present value of tax shield = Rs.1,022,076

5.
(a) Base case BPV

= - 8,000,000 + 2,000,000 x PVIFA (18,6)


= - Rs.1,004,000

(b) Adjusted NPV after adjustment for issue cost of external equity

= Base case NPV Issue cost


= - 1,004,000 [ 3,000,000 / 0.9 3,000,000]
= - Rs.1,337,333

65
(c) The present value of interest tax shield is calculated below :

Year Debt outstanding at Interest Tax shield Present value of


the beginning tax shield
1 5,000,000 750,000 300,000 260,880
2 5,000,000 750,000 300,000 226,830
3 4,000,000 600,000 240,000 157,800
4 3,000,000 450,000 180,000 102,924
5 2,000,000 300,000 120,000 59,664
6 1,000,000 150,000 60,000 25,938

Present value of tax shield = Rs.834,036

66
Chapter 18
RAISING LONG TERM FINANCE

1 Underwriting Shares Excess/ Credit Net


commitment procured shortfall shortfall

A 70,000 50,000 (20,000) 4919 (15081)

B 50,000 30,000 (20,000) 3514 (16486)

C 40,000 30,000 (10,000) 2811 (7189)

D 25,000 12,000 (13,000) 1757 (11243)

E 15,000 28,000 13,000

2.
Underwriting Shares Excess/ Credit Net
commitment procured Shortfall shortfall

A 50,000 20,000 (30,000) 14286 (15714)

B 20,000 10,000 (10,000) 5714 (4286)

C 30,000 50,000 20,000 - -

3. Po = Rs.220 S = Rs.150 N=4


a. The theoretical value per share of the cum-rights stock would simply be
Rs.220

b. The theoretical value per share of the ex-rights stock is :

67
NPo+S 4 x 220 +150
= = Rs.206
N+1 4+1

c. The theoretical value of each right is :


Po S 220 150
= = Rs.14
N+1 4+1
The theoretical value of 4 rights which are required to buy 1 share is Rs.14x14=Rs.56.

4. Po = Rs.180 N=5
a. The theoretical value of a right if the subscription price is Rs.150
Po S 180 150
= = Rs.5
N+1 5+1

b. The ex-rights value per share if the subscription price is Rs.160


NPo + S 5 x 180 + 160
= = Rs.176.7
N+1 5+1

c. The theoretical value per share, ex-rights, if the subscription price is


Rs.180? 100?
5 x 180 + 180
= Rs.180
5+1

5 x 180 + 100
= Rs.166.7
5+1

68
Chapter 19
CAPITAL STRUCTURE AND FIRM VALUE

1. Net operating income (O) : Rs.30 million


Interest on debt (I) : Rs.10 million
Equity earnings (P) : Rs.20 million
Cost of equity (rE) : 15%

Cost of debt (rD) : 10%


Market value of equity (E) : Rs.20 million/0.15 =Rs.133 million
Market value of debt (D) : Rs.10 million/0.10 =Rs.100 million
Market value of the firm (V) : Rs.233 million

2. Box Cox

Market value of equity 2,000,000/0.15 2,000,000/0.15


= Rs.13.33 million = Rs.13.33 million
Market value of debt 0 1,000,000/0.10
=Rs.10 million
Market value of the firm Rs.13.33million =23.33 million

(a) Average cost of capital for Box Corporation


13.33. 0
x 15% + x 10% = 15%
13.33 13.33

Average cost of capital for Cox Corporation


13.33 10.00
x 15% + x 10% = 12.86%
23.33 23.33

(b) If Box Corporation employs Rs.30 million of debt to finance a project that yields
Rs.4 million net operating income, its financials will be as follows.

Net operating income Rs.6,000,000


Interest on debt Rs.3,000,000
Equity earnings Rs.3,000,000
Cost of equity 15%

69
Cost of debt 10%
Market value of equity Rs.20 million
Market value of debt Rs.30 million
Market value of the firm Rs.50 million

Average cost of capital


20 30
15% x + 10% = 12%
50 50

(c) If Cox Corporation sells Rs.10 million of additional equity to retire


Rs.10 million of debt , it will become an all-equity company. So its
average cost of capital will simply be equal to its cost of equity,
which is 15%.

3. rE = rA + (rA-rD)D/E
20 = 12 + (12-8) D/E
So D/E = 2

4. E D E D
rE rD rA = rE + rD
D+E D+E (%) (%) D+E D+E

1.00 0.00 11.0 6.0 11.00


0.90 0.10 11.0 6.5 10.55
0.80 0.20 11.5 7.0 10.60
0.70 0.30 12.5 7.5 11.00
0.60 0.40 13.0 8.5 11.20
0.50 0.50 14.0 9.5 11.75
0.40 0.60 15.0 11.0 12.60
0.30 0.70 16.0 12.0 13.20
0.20 0.80 18.0 13.0 14.00
0.10 0.90 20.0 14.0 14.20

The optimal debt ratio is 0.10 as it minimises the weighted average


cost of capital.

5. (a) If you own Rs.10,000 worth of Bharat Company, the levered company
which is valued more, you would sell shares of Bharat Company, resort
to personal leverage, and buy the shares of Charat Company.
(b) The arbitrage will cease when Charat Company and Bharat Company
are valued alike

70
6. The value of Ashwini Limited according to Modigliani and Miller
hypothesis is
Expected operating income 15
= = Rs.125 million
Discount rate applicable to the 0.12
risk class to which Aswini belongs
7. The average cost of capital(without considering agency and bankruptcy cost)
at various leverage ratios is given below.

D E E D
rD rE rA = rE + rD
D+E D+ E % % D+E D+E
(%)

0 1.00 4.0 12.0 12.0


0.10 0.90 4.0 12.0 11.2
0.20 0.80 4.0 12.5 10.8
0.30 0.70 4.0 13.5 10.36
0.40 0.60 4.0 13.5 9.86
0.50 0.50 4.0 14.0 9.30
0.60 0.40 4.0 14.5 8.68
0.70 0.30 4.0 15.0 8.14
0.80 0.20 4.0 15.5 7.90
0.90 0.10 4.0 16.0 7.72 Optimal

b. The average cost of capital considering agency and bankruptcy costs is


given below
.
D E E D
rD rE rA = rE + rD
D+E D+ E % % D+E D+E
(%)

0 1.00 4.0 12.0 12.0


0.10 0.90 4.0 12.0 11.2
0.20 0.80 4.0 13.0 11.2
0.30 0.70 4.2 14.0 11.06
0.40 0.60 4.4 15.0 10.76
0.50 0.50 4.6 16.0 10.30
0.60 0.40 4.8 17.0 9.68
0.70 0.30 5.2 18.0 9.04
0.80 0.20 6.0 19.0 8.60
0.90 0.10 6.8 20.0 8.12 Optimal
8. The tax advantage of one rupee of debt is :

71
1-(1-tc) (1-tpe) (1-0.55) (1-0.05)
= 1 -
(1-tpd) (1-0.25)

= 0.43 rupee

Chapter 20
CAPITAL STRUCTURE DECISION

1.(a) Currently
No. of shares = 1,500,000
EBIT = Rs 7.2 million
Interest = 0
Preference dividend = Rs.12 x 50,000 = Rs.0.6 million
EPS = Rs.2

(EBIT Interest) (1-t) Preference dividend


EPS =
No. of shares

(7,200,000 0 ) (1-t) 600,000


Rs.2 =
1,500,000

Hence t = 0.5 or 50 per cent

The EPS under the two financing plans is :

Financing Plan A : Issue of 1,000,000 shares

(EBIT - 0 ) ( 1 0.5) - 600,000


EPSA =
2,500,000

Financing Plan B : Issue of Rs.10 million debentures carrying 15 per cent


interest

(EBIT 1,500,000) (1-0.5) 600,000


EPSB =
1,500,000

The EPS EBIT indifference point can be obtained by equating EPSA and EPSB

(EBIT 0 ) (1 0.5) 600,000 (EBIT 1,500,000) (1 0.5) 600,000

72
=
2,500,000 1,500,000

Solving the above we get EBIT = Rs.4,950,000 and at that EBIT, EPS is Rs.0.75
under both the plans

(b) As long as EBIT is less than Rs.4,950,000 equity financing maximixes EPS.
When EBIT exceeds Rs.4,950,000 debt financing maximises EPS.

2.
(a) EPS EBIT equation for alternative A
EBIT ( 1 0.5)
EPSA =
2,000,000
(b) EPS EBIT equation for alternative B
EBIT ( 1 0.5 ) 440,000
EPSB =
1,600,000

(c) EPS EBIT equation for alternative C


(EBIT 1,200,000) (1- 0.5)
EPSC =
1,200,000

(d) The three alternative plans of financing ranked in terms of EPS over varying
Levels of EBIT are given the following table

Ranking of Alternatives

EBIT EPSA EPSB EPSC


(Rs.) (Rs.) (Rs.) (Rs.)

2,000,000 0.50(I) 0.35(II) 0.33(III)


2,160,000 0.54(I) 0.40(II) 0.40(II)
3,000,000 0.75(I) 0.66(II) 0.75(I)
4,000,000 1.00(II) 0.98(III) 1.17(I)
4,400,000 1.10(II) 1.10(II) 1.33(I)
More than 4,400,000 (III) (II) (I)

3. Plan A : Issue 0.8 million equity shares at Rs. 12.5 per share.
Plan B : Issue Rs.10 million of debt carrying interest rate of 15 per cent.

(EBIT 0 ) (1 0.6)
EPSA =

73
1,800,000
(EBIT 1,500,000) (1 0.6)
EPSB =
1,000,000

Equating EPSA and EPSB , we get


(EBIT 0 ) (1 0.6) (EBIT 1,500,000) (1 0.6)
=
1,800,000 1,000,000

Solving this we get EBIT = 3,375,000 or 3.375 million

Thus the debt alternative is better than the equity alternative when
EBIT > 3.375 million

EBIT EBIT 3.375 7.000


Prob(EBIT>3,375,000) = Prob >
EBIT 3.000

= Prob [z > - 1.21]


= 0.8869

4. ROE = [ ROI + ( ROI r ) D/E ] (1 t )


15 = [ 14 + ( 14 8 ) D/E ] ( 1- 0.5 )
D/E = 2.67

5. ROE = [12 + (12 9 ) 0.6 ] (1 0.6)


= 5.52 per cent

6. 18 = [ ROI + ( ROI 8 ) 0.7 ] ( 1 0.5)


ROI = 24.47 per cent
EBIT
7. a. Interest coverage ratio =
Interest on debt

150
=
40
= 3.75
EBIT + Depreciation
b. Cash flow coverage ratio =
Loan repayment instalment

74
Int.on debt +
(1 Tax rate)
= 150 + 30

40 + 50

= 2
8. The debt service coverage ratio for Pioneer Automobiles Limited is given by :
5
PAT i + Depi + Inti)
i=1
DSCR = 5
Inti + LRIi)
i=1

= 133.00 + 49.14 +95.80

95.80 + 72.00

= 277.94
167.80
= 1.66

9. (a) If the entire outlay of Rs. 300 million is raised by way of debt carrying 15 per cent
interest, the interest burden will be Rs. 45 million.
Considering the interest burden the net cash flows of the firm during
a recessionary year will have an expected value of Rs. 35 million (Rs.80 million - Rs. 45
million ) and a standard deviation of Rs. 40 million .
Since the net cash flow (X) is distributed normally
X 35

40
has a standard normal deviation
Cash flow inadequacy means that X is less than 0.
0.35
Prob(X<0) = Prob (z< ) = Prob (z<- 0.875)
40
= 0.1909

(b) Since = Rs.80 million, = Rs.40 million , and the Z value corresponding to the risk
tolerance limit of 5 per cent is 1.645, the cash available from the operations to service the
debt is equal to X which is defined as :
X 80
= - 1.645

75
40
X = Rs.14.2 million
Given 15 per cent interest rate, the debt than be serviced is
14.2
= Rs. 94.67 million
0.15

Chapter 21
DIVIDEND POLICY AND FIRM VALUE

1. Payout ratio Price per share

3(0.5)+3(0.5) 0.15
0.5
0.12
= Rs. 28.13
0.12

3(0.7 5)+3(0.25) 0.15


0.12
0.75 = Rs. 26.56
0.12

3(1.00)
1.00 = Rs. 25.00
0.12

2. Payout ratio Price per share

8(0.25)
0.25 = undefined
0.12 0.16(0.75)
8(0.50)

0.50 = Rs.100
0.12 0.16(0.50)
8(1.00)
1.0 =Rs.66.7
0.12 0.16 (0)

76
3.
P Q
Next years price 80 74
Dividend 0 6
Current price P Q
Capital appreciation (80-P) (74-Q)
Post-tax capital appreciation 0.9(80-P) 0.9 (74-Q)
Post-tax dividend income 0 0.8 x 6
Total return 0.9 (80-P) 0.9 (74-Q) + 4.8
P Q
= 14% =14%
Current price (obtained by solving P = Rs.69.23 Q = Rs.68.65
the preceding equation)

77
Chapter 22
DIVIDEND DECISION

1. a. Under a pure residual dividend policy, the dividend per share over the 4 year
period will be as follows:

DPS Under Pure Residual Dividend Policy


( in Rs.)

Year 1 2 3 4

Earnings 10,000 12,000 9,000 15,000


Capital expenditure 8,000 7,000 10,000 8,000
Equity investment 4,000 3,500 5,000 4,000
Pure residual
dividends 6,000 8,500 4,000 11,000
Dividends per share 1.20 1.70 0.80 2.20

b. The external financing required over the 4 year period (under the assumption that the
company plans to raise dividends by 10 percents every two years) is given below :
Required Level of External Financing
(in Rs.)

Year 1 2 3 4

A. Net income 10,000 12,000 9,000 15,000

B. Targeted DPS 1.00 1.10 1.10 1.21

C. Total dividends 5,000 5,500 5,500 6,050

D. Retained earnings(A-C) 5,000 6,500 3,500 8,950

E. Capital expenditure 8,000 7,000 10,000 8,000

78
F. External financing
requirement 3,000 500 6,500 Nil
(E-D)if E > D or 0 otherwise

c. Given that the company follows a constant 60 per cent payout ratio, the dividend per share
and external financing requirement over the 4 year period are given below

Dividend Per Share and External Financing Requirement


(in Rs.)

Year 1 2 3 4

A. Net income 10,000 12,000 9,000 15,00

B. Dividends 6,000 7,200 5,400 9,000

C. Retained earnings 4,000 4,800 3,600 6,000

D. Capital expenditure 8,000 7,000 10,000 8,000

E. External financing
(D-C)if D>C, or 0 4,000 2,200 6,400 2,000
otherwise

F. Dividends per share 1.20 1.44 1.08 1.80

2. Given the constraints imposed by the management, the dividend per share has to
be between Rs.1.00 (the dividend for the previous year) and Rs.1.60 (80 per
cent of earnings per share)
Since share holders have a preference for dividend, the dividend should be
raised over the previous dividend of Rs.1.00 . However, the firm has substantial
investment requirements and it would be reluctant to issue additional equity
because of high issue costs ( in the form of underpricing and floatation costs)
Considering the conflicting requirements, it seems to make sense to pay
Rs.1.20 per share by way of dividend. Put differently the pay out ratio may be
set at 60 per cent.

3. According to the Lintner model


Dt = cr EPSt + (1-c)Dt 1
EPSt =3.00, c= 0.7, r=0.6 , and Dt-1
Hence
Dt = 0.7 x 0.6 x 3.00 + (1-0.7)1.20

79
= Rs.1.62

4. The bonus ratio (b) must satisfy the following constraints :


(R-Sb)0.4S (1+b) (1)
0.3 PBT 0.1 S(1+b) (2)

R = Rs.100 million, S= Rs.60 million, PBT = Rs.60 million


Hence the constraints are
(100-60 b) 0.4 x 60 (1+b) (1a)
0.3 x 600.1 x 60 (1+b) (2a)

These simplify to
b 76/84
b 2

The condition b 76/84 is more restructive than b 2


So the maximum bonus ratio is 76/84 or 19/21

80
Chapter 23
Debt Analysis and Management

1. (i) Initial Outlay


(a) Cost of calling the old bonds
Face value of the old bonds 250,000,000
Call premium 15,000,000
265,000,000
(b) Net proceeds of the new bonds
Gross proceeds 250,000,000
Issue costs 10,000,000

240,000,000
(c) Tax savings on tax-deductible expenses
Tax rate[Call premium+Unamortised issue cost on
the old bonds] 9,200,000
0.4 [ 15,000,000 + 8,000,000]
Initial outlay i(a) i(b) i(c) 15,800,000

(ii) Annual Net Cash Savings


(a) Annual net cash outflow on old bonds
Interest expense 42,500,000
- Tax savings on interest expense and amortisation of
issue expenses 17,400,000
0.4 [42,500,000 + 8,000,000/10]
25,100,000
(b) Annual net cash outflow on new bonds
Interest expense 37,500,000
- Tax savings on interest expense and amortisation of
issue cost 15,500,000
0.4 [ 37,500,000 10,000,000/8]
22,000,000
Annual net cash savings : ii(a) ii(b) 3,100,000

(iii) Present Value of the Annual Cash Savings


Present value of an 8-year annuity of 3,100,000 at a
discount rate of 9 per cent which is the post tax cost

81
of new bonds 3,100,000 x 5.535 17,158,500

(iv) Net Present Value of Refunding the Bonds


(a) Present value of annual cash savings 17,158,500
(b) Net initial outlay 15,800,000
(c) Net present value of refunding the bonds :
iv(a) iv(b). 1,358,500
2. (i) Initial Outlay
(a) Cost of calling the old bonds
Face value of the old bonds 120,000,000
Call premium 4,800,000

124,800,000
(b) Net proceeds of the new issue
Gross proceeds 120,000,000
Issue costs 2,400,000

117,600,000
(c) Tax savings on tax-deductible expenses 3,120,000
Tax rate[Call premium+Unamortised issue costs on
the old bond issue]
0.4 [ 4,800,000 + 3,000,000]
Initial outlay i(a) i(b) i(c) 4,080,000

(ii) Annual Net Cash Savings


(a) Annual net cash out flow on old bonds
Interest expense 19,200,000
- Tax savings on interest expense and amortisation of
issue costs 7,920,000
0.4[19,200,000 + 3,000,000/5]
11,280,000

(b) Annual net cash outflow on new bonds


Interest expense 18,000,000
- Tax savings on interest expense and amortistion of issue
costs 7,392,000
0.4[18,000,000 + 2,400,000/5]
10,608,000
Annual net cash savings : ii(a) ii(b) 672,000

(iii) Present Value of the Annual Net Cash Savings


Present value of a 5 year annuity of 672,000 at
as discount rate of 9 per cent, which is the post-tax 2,614,080 cost of
new bonds

82
(iv) Net Present Value of Refunding the Bonds
(a) Present value of annual net cash savings 2,614,080
(b) Initial outlay 4,080,000
(c) Net present value of refunding the bonds : - 1,466,000
iv(a) iv(b)

3. Yield to maturity of bond P


8 160 1000
918.50 = +
t=1 (1+r)t (1+r)8

r or yield to maturity is 18 percent

Yield to maturity of bond Q


5 120 1000
761 = +
t=1 (1+r)t (1+r)5

r or yield to maturity is 20 per cent

Duration of bond P is calculated below

Year Cash flow Present Value Proportion of Proportion of bonds


at 18% bonds value Value x Time

1 160 135.5 0.148 0.148


2 160 114.9 0.125 0.250
3 160 97.4 0.106 0.318
4 160 82.6 0.090 0.360
5 160 69.9 0.076 0.380
6 160 59.2 0.064 0.384
7 160 50.2 0.055 0.385
8 160 308.6 0.336 2.688

4.913

Duration of bond Q is calculated below

Year Cash flow Present Value Proportion of Proportion of bonds


at 20% bonds value Value x Time

1 120 100.0 0.131 0.131


2 120 83.2 0.109 0.218

83
3 120 69.5 0.091 0.273
4 120 57.8 0.076 0.304
5 1120 450.2 0.592 2.960

3.886

Volatility of bond P Volatility of bond Q


4.913 3.886
= 4.16 = 3.24
1.18 1.20

4. The YTM for bonds of various maturities is

Maturity YTM(%)

1 12.36

2 13.10

3 13.21

4 13.48

5 13.72

Graphing these YTMs against the maturities will give the yield curve

The one year treasury bill rate , r1, is

1,00,000
- 1 = 12.36 %
89,000

To get the forward rate for year 2, r2, the following equation may be set up :

12500 112500
99000 = +
(1.1236) (1.1236)(1+r2)

Solving this for r2 we get r2 = 13.94%

To get the forward rate for year 3, r3, the following equation may be set up :

84
13,000 13,000 113,000
99,500 = + +
(1.1236) (1.1236)(1.1394) (1.1236)(1.1394)(1+r3)

Solving this for r3 we get r3 = 13.49%

To get the forward rate for year 4, r4 , the following equation may be set up :

13,500 13,500 13,500


100,050 = + +
(1.1236) (1.1236)(1.1394) (1.1236)(1.1394)(1.1349)

113,500
+
(1.1236)(1.1394)(1.1349)(1+r4)

Solving this for r4 we get r4 = 14.54%

To get the forward rate for year 5, r5 , the following equation may be set up :

13,750 13,750 13,750


100,100 = + +
(1.1236) (1.1236)(1.1394) (1.1236)(1.1394)(1.1349)

13,750
+
(1.1236)(1.1394)(1.1349)(1.1454)

113,750
+
(1.1236)(1.1394)(1.1349)(1.1454)(1+r5)

Solving this for r5 we get r5 = 15.08%

85
Chapter 25
HYBRID FINANCING

1. The product of the standard deviation and square root of time is :


t = 0.35 2 = 0.495
The ratio of the stock price to the present value of the exercise price is :

Stock price 40
= = 1.856
PV (Exercise price) 25/(1.16)

The ratio of the value of call option to stock price corresponding to numbers
0.495 and 1.856 can be found out from Table A.6 by interpolation. Note the
table gives values for the following combinations

1.75 2.00

0.45 44.6 50.8

0.50 45.3 51.3

Since we are interested in the combination 0.495 and 1.856 we first interpolate
between 0.450 and 0.500 and then interpolate between 1.75 and 2.00

Interpolation between 0.450 and 0.500 gives

1.75 2.00

0.450 44.6 50.8

0.495 45.23 51.25

0.500 45.3 51.3

Then, interpolation between 1.75 and 2.00 gives

86
1.75 1.856 2.00

0.495 45.23 47.78 51.25

Chapter 24
LEASING, HIRE PURCHASE, AND PROJECT FINANCE

1. NPV of the Purchase Option


(Rs.in 000)
Year 0 1 2 3 4 5
1.Investment(I) (1,500)
2.Revenues(Rt) 1,700 1,700 1,700 1,700 1,700
3.Costs(other than
(Depreciation)(Ct) 900 900 900 900 900
4.Depreciation(Dt) 500 333.3 222.2 148.1 98.8
5.Profit before tax
(Rt-Ct-Dt) 300 466.7 577.8 651.9 701.2
6.Profit after tax: 5(1-t) 210 326.7 404.5 456.3 490.8
7.Net salvage value 300
8.Net cash flow
(1+6+4+7) (1,500) 710 610 626.7 604.4 889.6
9.Discount factor
at 11 percent 1.000 0.901 0.812 0.731 0.659 0.593
10.Present value (8x9) (1,500) 639.7 495.3 458.1 398.3 527.5

NPV(Purchases)= - 1500+639.7+495.3+458.1+398.3+527.5 = 1018.9

NPV of the Leasing Option


(Rs. in 000)
Year 0 1 2 3 4 5
1.Revenues(Rt) - 1,700 1,700 1,700 1,700 1,700
2.Costs(other than
lease rentals)(Ct) 900 900 900 900 900
3.Lease rentals(Lt) 420 420 420 420 420 0
4.Profit before tax
(Rt-Ct-Lt) -420 380 380 380 380 800
5.Profit after tax (which
also reflects the net
cash flow)(1-t) -294 266 266 266 266 560
6.Discount factor at

87
13 per cent 1.000 0.885 0.783 0.693 0.613 0.543
7.Present value(5x6) -294 -235.4 208.3 184.3 163.1 304.1

NPV(Leasing) = -294+235.4+208.3+184.3+163.1+304.1 = 801.2

2. Under the hire purchase proposal the total interest payment is


2,000,000 x 0.12 x 3 = Rs. 720,000

The interest payment of Rs. 720,000 is allocated over the 3 years period using
the sum of the years digits method as follows:
Year Interest allocation

366
1 x Rs.720,000 = Rs.395,676
666

222
2 x Rs.720,000 = Rs.240,000
666

78
3 x Rs.720,000 = Rs.84,324
666

The annual hire purchase instalments will be :

Rs.2,000,000 + Rs.720,000
= Rs.906,667
3

The annual hire purchase instalments would be split as follows

Year Hire purchase instalment Interest Principal repayment


1 Rs.906,667 Rs.395,676 Rs. 510,991
2 Rs.906,667 Rs.240,000 Rs. 666,667
3 Rs.906,667 Rs. 84,324 Rs. 822,343

The lease rental will be as follows :


Rs. 560,000 per year for the first 5 years
Rs. 20,000 per year for the next 5 years

88
The cash flows of the leasing and hire purchse options are shown below

Year Leasing High Purchase -It(1-tc)-PRt+


- LRt (1-tc) -It(1-tc) -PRt Dt(tc) NSVt Dt(tc)+NSVt

1 -560,000(1-.4)=-336,000 -395,676(1-.4) -510,991 500,000(0.4) -548,397


2 -560,000(1-.4)=-336,000 -240,000(1-.4) -666,667 375,000(0.4) -660,667
3 -560,000(1-.4)=-336,000 - 84,324(1-.4) -822,343 281,250(0.4) -760,437
4 -560,000(1-.4)=-336,000 210,938(0.4) 84,375
5 -560,000(1-.4)=-336,000 158,203(0.4) 63,281
6 - 20,000(1-.4)= - 12,000 118,652(0.4) 47,461
7 - 20,000(1-.4)= - 12,000 88,989(0.4) 35,596
8 - 20,000(1-.4)= - 12,000 66,742(0.4) 26,697
9 - 20,000(1-.4)= - 12,000 50,056(0.4) 20,023
10 - 20,000(1-.4)= - 12,000 37,542(0.4) 200,000 215,017

Present value of the leasing option

5 336,000 10 12,000
= - = - 1,302,207
t=1 (1.10)t t=6 (1.10) t

Present value of the hire purchase option

548,397 660,667 760,437 84,375


=- - - -
(1.10) (1.10)2 (1.10)3 (1.10)4

63,281 47,461 35,596 26,697


+ + +
(1.10)5 (1.10)6 (1.10)7 (1.10)8

20,023 215,017
+
(1.10.9 (1.10)10

89
= - 1,369,383

Since the leasing option costs less than the hire purchase option , Apex should choose the
leasing option.

Chapter 26
WORKING CAPITAL POLICY

Average inventory
1 Inventory period =
Annual cost of goods sold/365

(60+64)/2
= = 62.9 days
360/365

Average accounts receivable


Accounts receivable =
period Annual sales/365

(80+88)/2
= = 61.3 days
500/365

Average accounts payable


Accounts payable =
period Annual cost of goods sold/365

(40+46)/2
= = 43.43 days
360/365

Operating cycle = 62.9 + 61.3 = 124.2 days


Cash cycle = 124.2 43.43 = 80.77 days

(110+120)/2
2. Inventory period = = 56.0 days
750/365

(140+150)/2
Accounts receivable = = 52.9 days
period 1000/365

90
(60+66)/2
Accounts payable = = 30.7 days
period 750/365

Operating cycle = 56.0 + 52.9 = 108.9 days


Cash cycle = 108.9 30.7 = 78.2 days
Rs.
3. 1. Sales 3,600,000
Less : Gross profit (25 per cent) 900,000
Total manufacturing cost 2,700,000
Less : Materials 900,000
Wages 720,000 1,620,000
Manufacturing expenses 1,080,000

2. Cash manufacturing expenses 960,000


(80,000 x 12)
3. Depreciation : (1) (2) 120,000
4. Total cash cost
Total manufacturing cost 2,700,000
Less: Depreciation 120,000
Cash manufacturing cost 2,580,000
Add: Administration and sales
promotion expenses 360,000

2,940,000

A : Current Assets Rs.

Total cash cost 2,940,000


Debtors x 2 = x 2= 490,000
12 12

Material cost 900,000


Raw material x 1 = x 1= 75,000
stock 12 12

Cash manufacturing cost 2,580,000


Finished goods x1= x 1= 215,000
stock 12 12

Cash balance A predetermined amount = 100,000

91
Sales promotion expenses 120,000
Prepaid sales x 1.5 = x 1.5 = 15,000
promotion 12 12
expenses
Cash balance A predetermined amount = 100,000

A : Current Assets = 995,000

B : Current Liabilites Rs.

Material cost 900,000


Sundry creditors x 2= x 2 = 150,000
12 12

Manufacturing One months cash


expenses outstanding manufacturing expenses = 80,000

Wages outstanding One months wages = 60,000

B : Current liabilities 290,000

Working capital (A B) 705,000


Add 20 % safety margin 141,000
Working capital required 846,000

92
Chapter 27
CASH AND LIQUIDITY MANAGEMENT

1. The forecast of cash receipts, cash payments, and cash position is prepared in the
statements given below

Forecast of Cash Receipts (Rs. in 000s)

November December January February March April May June

1. Sales 120 120 150 150 150 200 200 200


2. Credit sales 84 84 105 105 105 140 140 140
3. Cash sales 36 36 45 45 45 60 60 60
4. Collection of receivables
(a) Previous month 33.6 33.6 42.0 42.0 42.0 56.0 56.0
(b) Two months earlier 50.4 50.4 63.0 63.0 63.0 84.0
5. Sale of machine 70.0
6. Interest on securities 3.0
7. Total receipts 129.0 137.4 150.0 235.0 179.0 203.0
(3+4+5+6)

Forecast of Cash Payments (Rs. in 000s)

December January February March April May June

1. Purchases 60 60 60 60 80 80 80
2. Payment of accounts 60 60 60 60 80 80
payable
3. Cash purchases 3 3 3 3 3 3
4. Wage payments 25 25 25 25 25 25
5. Manufacturing
expenses 32 32 32 32 32 32
6. General, administrative
& selling expenses 15 15 15 15 15 15
7. Dividends 30
8. Taxes 35

93
9. Acquisition of
machinery 80

Total payments(2to9) 135 135 215 135 155 220

Summary of Cash Forecast (Rs.in 000s)

January February March April May June

1. Opening balance 28
2. Receipts 129.0 137.4 150.0 235.0 179.0 203.0
3. Payments 135.0 135.0 215.0 135.0 155.0 220.0
4. Net cash flow(2-3) (6.0) 2.4 (65.0) 100.0 24.0 (17.0)
5. Cumulative net cash flow (6.0) (3.6) (68.6) 31.4 55.4 (38.4)
6. Opening balance +
Cumulative net cash flow 22.0 24.4 (40.6) 59.4 83.4 66.4
7. Minimum cash balance
required 30.0 30.0 30.0 30.0 30.0 30.0
8. Surplus/(Deficit) (8.0) (5.6) (70.6) 29.4 53.0 36.4

2. The projected cash inflows and outflows for the quarter, January through March, is shown
below .

Month December January February March


(Rs.) (Rs.) (Rs.) (Rs.)

Inflows :
Sales collection 50,000 55,000 60,000

Outflows :
Purchases 22,000 20,000 22,000 25,000
Payment to sundry creditors 22,000 20,000 22,000
Rent 5,000 5,000 5,000
Drawings 5,000 5,000 5,000
Salaries & other expenses 15,000 18,000 20,000
Purchase of furniture - 25,000 -

Total outflows(2to6) 47,000 73,000 52,000

94
Given an opening cash balance of Rs.5000 and a target cash balance of Rs.8000, the
surplus/deficit in relation to the target cash balance is worked out below :

January February March


(Rs.) (Rs.) (Rs.)

1. Opening balance 5,000


2. Inflows 50,000 55,000 60,000
3. Outflows 47,000 73,000 52,000
4. Net cash flow (2 - 3) 3,000 (18,000) 8,000
5. Cumulative net cash flow 3,000 (15,000) (7,000)
6. Opening balance + Cumulative
net cash flow 8,000 (10,000) (2,000)
7. Minimum cash balance required 8,000 8,000 8,000
8. Surplus/(Deficit) - (18,000) (10,000)

3. The balances in the books of Datta co and the books of the bank are shown below:

(Rs.)
1 2 3 4 5 6 7 8 9 10
Books of Datta
Co:

Opening 30,00 46,00 62,00 78,000 1,10,00 1,26,0 1,42,0 1,58,0 1,74,0
Balance 0 0 0 94,000 0 00 00 00 00
Add: Cheque 20,00 20,00 20,00 20,000
received 0 0 0 20,000 20,000 20,000 20,000 20,000 20,000
Less: Cheque 4,000
issued 4,000 4,000 4,000 4,000 4,000 4,000 4,000 4,000 4,000
Closing 46,00 62,00 78,00 94,000 1,10,0 1,26,00 1,42,0 1,58,0 1,74,0 1,90,0
Balance 0 0 0 00 0 00 00 00 00

Books of the
Bank:

Opening 30,00 30,00 30,00 30,000 30,000 30,000 50,000 70,000 1,06,0
Balance 0 0 0 90,000 00
Add: Cheques - - - - - 20,000 20,000 20,000

95
realised 20,000 20,000
Less: Cheques - - - - - - - -
debited 4,000 4,000
Closing 30,00 30,00 30,00 30,000 30,000 50,000 70,000 90,000 1,06,0 1,22,0
Balance 0 0 0 00 00

From day 9 we find that the balance as per the banks books is less than the balance as per Datta
Companys books by a constant sum of Rs.68,000. Hence in the steady situation Datta Company has
a negative net float of Rs.68,000.

4. Optimal conversion size is


2bT
C =
I
b = Rs.1200, T= Rs.2,500,000, I = 5% (10% dividend by two)

So,
2 x 1200 x 2,500,000
C = = Rs.346,410
0.05

5.
3 3 b2
RP = + LL
4I

UL = 3 RP 2 LL

I = 0.12/360 = .00033, b = Rs.1,500, = Rs.6,000, LL = Rs.100,000

3 3 x 1500 x 6,000 x 6,000


RP = + 100,000
4 x .00033

= 49,695 + 100,000 = Rs.149,695

UL = 3RP 2LL = 3 x 149,695 2 x 100,000


= Rs.249,085

96
Chapter 28
CREDIT MANAGEMENT

1. RI = [S(1-V)- Sbn](1-t)- k I
S
I = x ACP x V
360
S = Rs.10 million, V=0.85, bn =0.08, ACP= 60 days, k=0.15, t = 0.40

Hence, RI = [ 10,000,000(1-0.85)- 10,000,000 x 0.08 ] (1-0.4)

-0.15 x 10,000,000 x 60 x 0.85

360
= Rs. 207,500

2. RI = [S(1-V)- Sbn] (1-t) k I

So S
I = (ACPN ACPo) +V(ACPN)
360 360

S=Rs.1.5 million, V=0.80, bn=0.05, t=0.45, k=0.15, ACPN=60, ACPo=45, So=Rs.15 million
Hence RI = [1,500,000(1-0.8) 1,500,000 x 0.05] (1-.45)

97
-0.15 (60-45) 15,000,000 + 0.8 x 60 x 1,500,000

360 360
= 123750 123750 = Rs. 0

3. RI = [S(1-V) DIS ] (1-t) + k I


DIS = pn(So+S)dn poSodo

So S
I = (ACPo-ACPN) - x ACPN x V
360 360

So =Rs.12 million, ACPo=24, V=0.80, t= 0.50, r=0.15, po=0.3, pn=0.7,


ACPN=16, S=Rs.1.2 million, do=.01, dn= .02
Hence
RI = [ 1,200,000(1-0.80)-{0.7(12,000,000+1,200,000).02-
0.3(12,000,000).01}](1-0.5)

12,000,000 1,200,000
+ 0.15 (24-16) - x 16 x 0.80
360 360

= Rs.79,200

4. RI = [S(1-V)- BD](1-t) k I
BD=bn(So+S) boSo

So S
I = (ACPN ACPo) + x ACPN x V
360 360

So=Rs.50 million, ACPo=25, V=0.75, k=0.15, bo=0.04, S=Rs.6 million,


ACPN=40 , bn= 0.06 , t = 0.3

RI = [ Rs.6,000,000(1-.75) {.06(Rs.56,000,000)-.04(Rs.50,000,000)](1-0.3)

Rs.50,000,000 Rs.6,000,000
- 0.15 (40-25) + x 40 x 0.75
360 360

98
= - Rs.289.495

5. 30% of sales will be collected on the 10th day


70% of sales will be collected on the 50th day
ACP = 0.3 x 10 + 0.7 x 50 = 38 days

Rs.40,000,000
Value of receivables = x 38
360

= Rs.4,222,222
Assuming that V is the proportion of variable costs to sales, the investment in
receivables is :
Rs.4,222,222 x V

6. 30% of sales are collected on the 5th day and 70% of sales are collected on the
25th day. So,
ACP = 0.3 x 5 + 0.7 x 25 = 19 days

Rs.10,000,000
Value of receivables = x 19
360

= Rs.527,778
Investment in receivables = 0.7 x 527,778
= Rs.395,833

7. Since the change in credit terms increases the investment in receivables,


RI = [S(1-V)- DIS](1-t) kI
So=Rs.50 million, S=Rs.10 million, do=0.02, po=0.70, dn=0.03,pn=0.60,
ACPo=20 days, ACPN=24 days, V=0.85, k=0.12 , and t = 0.40
DIS = 0.60 x 60 x 0.03 0.70 x 50 x 0.2
= Rs.0.38 million

50 10
I= (24-20) + x 24 x 0.85
360 360

= Rs.1.2222 million
RI = [ 10,000,000 (1-.85) 380,000 ] (1-.4) 0.12 x 1,222,222
= Rs.525,333

99
8. The decision tree for granting credit is as follows :

Customer pays(0.95)
Grant credit Profit 1500
Customer pays(0.85)
Grant credit Customer defaults(0.05)
Profit 1500 Refuse credit
Loss 8500
Customer defaults(0.15)
Loss 8500
Refuse credit

The expected profit from granting credit, ignoring the time value of money, is :

Expected profit on + Probability of payment x Expected profit on


Initial order and repeat order repeat order

{ 0.85(1500)-0.15(8500)} + 0.85 {0.95(1500)-.05(8500)}


= 0 + 850 = Rs.850

9. Profit when the customer pays = Rs.10,000 - Rs.8,000 = Rs.2000


Loss when the customer does not pay = Rs.8000
Expected profit = p1 x 2000 (1-p1)8000
Setting expected profit equal to zero and solving for p1 gives :
p1 x 2000 (1- p1)8000 = 0 p1 = 0.80
Hence the minimum probability that the customer must pay is 0.80

MINICASE
Solution:

Present Data
Sales : Rs.800 million
Credit period : 30 days to those deemed eligible
Cash discount : 1/10, net 30
Proportion of credit sales and cash sales are 0.7 and 0.3. 50 percent of the credit customers
avail of cash discount
Contribution margin ratio : 0.20
Tax rate : 30 percent
Post-tax cost of capital : 12 percent
ACP on credit sales : 20 days

100
Effect of Relaxing the Credit Standards on Residual Income

Incremental sales : Rs.50 million


Bad debt losses on incremental sales: 12 percent
ACP remains unchanged at 20 days

RI = [S(1 V) - Sbn] (1 t) R I

S
where I = x ACP x V
360

RI = [50,000,000 (1-0.8) 50,000,000 x 0.12] (1 0.3)

50,000,000
- 0.12 x x 20 x 0.8
360

= 2,800,000 266,667 = 2,533,333

Effect of Extending the Credit Period on Residual Income

RI = [S(1 V) - Sbn] (1 t) R I

So S
where I = (ACPn ACPo) + V (ACPn)
360 360

RI = [50,000,000 (1 0.8) 50,000,000 x 0] (1 0.3)

800,000,000 50,000,000
- 0.12 (50 20) x + 0.8 x 50 x
360 360

= 7,000,000 8,666,667
= - Rs.1,666,667

Effect of Relaxing the Cash Discount Policy on Residual Income

RI = [S (1 V) - DIS] (1 t) + R I
where I = savings in receivables investment
So S
= (ACPo ACPn) V x ACPn

101
360 360

800,000,000 20,000,000
= (20 16) 0.8 x x 16
360 360

= 8,888,889 711,111 = 8,177,778

DIS = increase in discount cost


= pn (So + S) dn po So do
= 0.7 (800,000,000 + 20,000,000) x 0.02 0.5 x 800,000,000 x 0.01
= 11,480,000 4,000,000 = 7,480,000

So, RI = [20,000,000 (1 0.8) 7,480,000] (1 0.3) + 0.12 x 8,177,778


= - 2,436,000 + 981,333
= - 1,454,667

Chapter 29
INVENTORY MANAGEMENT

1.
a. No. of Order Ordering Cost Carrying Cost Total Cost
Orders Per Quantity (U/Q x F) Q/2xPxC of Ordering
Year (Q) (where and Carrying
(U/Q) PxC=Rs.30)
Units Rs. Rs. Rs.

1 250 200 3,750 3,950


2 125 400 1,875 2,275
5 50 1,000 750 1,750
10 25 2,000 375 2,375

2 UF 2x250x200
b. Economic Order Quantity (EOQ) = =
PC 30
2UF = 58 units (approx)

102
2. a EOQ =
PC
U=10,000 , F=Rs.300, PC= Rs.25 x 0.25 =Rs.6.25

2 x 10,000 x 300
EOQ = = 980
6.25
10000
b. Number of orders that will be placed is = 10.20
980
Note that though fractional orders cannot be placed, the number of orders
relevant for the year will be 10.2 . In practice 11 orders will be placed during the year. However,
the 11th order will serve partly(to the extent of 20 percent) the present year and partly(to the
extent of 80 per cent) the following year. So only 20 per cent of the ordering cost of the 11th order
relates to the present year. Hence the ordering cost for the present year will be 10.2 x Rs.300

c. Total cost of carrying and ordering inventories


980
= [ 10.2 x 300 + x 6.25 ] = Rs.6122.5
2
3. U=6,000, F=Rs.400 , PC =Rs.100 x 0.2 =Rs.20

2 x 6,000 x 400
EOQ = = 490 units
20

U U Q(P-D)C Q* PC
= UD + - F- -
Q* Q 2 2

6,000 6,000
= 6000 x .5 + - x 400
490 1,000

1,000 (95)0.2 490 x 100 x 0.2


- -
2 2

= 30,000 + 2498 4600 = Rs.27898

4. U=5000 , F= Rs.300 , PC= Rs.30 x 0.2 = Rs.6

103
2 x 5000 x 300
EOQ = = 707 units
6
If 1000 units are ordered the discount is : .05 x Rs.30 = Rs.1.5 Change in
profit when 1,000 units are ordered is :

5,000 5,000
= 5000 x 1.5 + - x 300
707 1,000

1000 x 28.5 x 0.2 707 x 30 x 0.2


- - = 7500 + 622-729 =Rs.7393
2 2

If 2000 units are ordered the discount is : .10 x Rs.30 = Rs.3 Change in profit
when 2,000 units are ordered is :

5000 5000 2000x27x0.2 707x30x0.2


= 5000 x 3.0 + - x 300- -
707 2000 2 2

= 15,000 +1372 3279 = Rs.13,093

5. The quantities required for different combinations of daily usage rate(DUR)


and lead times(LT) along with their probabilities are given in the following
table

LT
(Days)
DUR 5(0.6) 10(0.2) 15(0.2)
(Units)

4(0.3) 20*(0.18) 40(0.06) 60(0.06)


6(0.5) 30 (0.30) 60(0.10) 90(0.10)
8(0.2) 40 (0.12) 80(0.04) 120(0.04)
*
Note that if the DUR is 4 units with a probability of 0.3 and the LT is 5 days with
a probability of 0.6, the requirement for the combination DUR = 4 units and LT =
5 days is 20 units with a probability of 0.3x0.6 = 0.18. We have assumed that the

104
probability distributions of DUR and LT are independent. All other entries in the
table are derived similarly.
The normal (expected) consumption during the lead time is :
20x0.18 + 30x0.30 + 40x0.12 + 40x0.06 + 60x0.10 + 80x0.04 + 60x0.06 + 90x0.10 +
120x0.04 = 46.4 tonnes

a. Costs associated with various levels of safety stock are given below :

Safety Stock Stock out Probability Expected Carrying Total Cost


Stock* outs(in Cost Stock out Cost
tonnes)

1 2 3 4 5 6 7
[3x4] [(1)x1,000] [5+6]

Tonnes Rs. Rs. Rs.


73.6 0 0 0 0 73,600 73,600
43.6 30 120,000 0.04 4,800 43,600 48,400

33.6 10 40,000 0.10


40 160,000 0.04 10,400 33,600 44,000

13.6 20 80,000 0.04


30 120,000 0.10 24,800 13,600 38,400
60 240,000 0.04

0 13.6 54,400 0.16


33.6 134,400 0.04 43,296 0 43,296
43.6 174,400 0.10
73.6 294,400

105
*
Safety stock = Maximum consumption during lead time Normal
consumption during lead time
So the optimal safety stock= 13.6 tonnes
Reorder level = Normal consumption during lead time + safety stock
K= 46.4 + 13.6 = 60 tonnes

b. Probability of stock out at the optimal level of safety stock = Probability


(consumption being 80 or 90 or 120 tonnes)

Probability (consumption = 80 tonnes) + Probability (consumption = 90 tonnes) +


Probability (consumption = 120 tonnes)
= 0.04 +0.10+0.04 = 0.18

6. Reorder point is given by the formula : S(L) + F SR (L)

= 30 x 40 + 2.00 30 x 1,000 x 40 = 3,391 units

7.
Item Annual Usage Price per Annual Ranking
(in Units) Unit Usage Value
Rs. Rs.

1 400 20.00 8,000 6


2 15 150.00 2,250 10
3 6,000 2.00 12,000 5
4 750 18.00 13,500 4
5 1,200 25.00 30,000 1
6 25 160.00 4,000 9
7 300 2.00 600 14
8 450 1.00 450 15
9 1,500 4.00 6,000 7
10 1,300 20.00 26,000 2
11 900 2.00 1,800 11
12 1,600 15.00 24,000 3
13 600 7.50 4,500 8
14 30 40.00 1,200 12
15 45 20.00 900 13

1,35,200

106
Cumulative Value of Items & Usage

Item Rank Annual Cumulative Cumulative Cumulative


No. UsageValue Annual Usage % of Usage % of Items
(Rs.) Value (Rs.) Value

5 1 30,000 30,000 22.2 6.7


10 2 26,000 56,000 41.4 13.3
12 3 24,000 80,000 59.2 20.0
4 4 13,500 93,500 69.2 26.7
3 5 12,000 105,500 78.0 33.3
1 6 8,000 113,500 83.9 40.0
9 7 6,000 119,500 88.4 46.7
13 8 4,500 124,000 91.7 53.3
6 9 4,000 128,000 94.7 60.0
2 10 2,250 130,250 96.3 66.7
11 11 1,800 132,050 97.7 73.3
14 12 1,200 133,250 98.6 80.0
15 13 900 134,150 99.2 86.7
7 14 600 134,750 99.7 93.3
8 15 450 135,200 100.0 100.0

107
Class No. of Items % to the Total Annual Usage % to Total Value
Value Rs.

A 4 26.7 93,500 69.2


B 3 20.0 26,000 19.2
C 18 53.3 15,700 11.6

15 135,200

Chapter 30
WORKING CAPITAL FINANCING

1. Annual interest cost is given by ,


Discount % 360
x
1- Discount % Credit period Discount period

Therefore, the annual per cent interest cost for the given credit terms will be as
follows:

a. 0.01 360
x = 0.182 = 18.2%
0.99 20

b. 0.02 360
x = 0.367 = 36.7%
0.98 20

c. 0.03 360
x = 0.318 = 31.8%
0.97 35

d. 0.01 360
x = 0.364 = 36.4%

108
0.99 10

2.
a.
0.01 360
x = 0.104 = 10.4%
0.99 35

b. 0.02 360
x = 0.21 = 21%
0.98 35

c. 0.03 360
x = 0.223 = 22.3%
0.97 50

d. 0.01 360
x = 0.145 = 14.5%
0.99 25
3. The maximum permissible bank finance under the three methods suggested by
The Tandon Committee are :

Method 1 : 0.75(CA-CL) = 0.75(36-12) = Rs.18 million


Method 2 : 0.75(CA)-CL = 0.75(36-12 = Rs.15 million
Method 3 : 0.75(CA-CCA)-CL = 0.75(36-18)-12 = Rs.1.5 million

109
Chapter 31
WORKING CAPITAL MANAGEMENT :EXTENSIONS

1.(a) The discriminant function is :

Zi = aXi + bYi
where Zi = discriminant score for the ith account
Xi = quick ratio for the ith account
Yi = EBDIT/Sales ratio for the ith account

The estimates of a and b are :


y2. dx - xy . dy
a =
x 2. y 2 - xy . xy

x 2. dy xy . dx
b =
x 2 y 2 xy xy

The basic calculations for deriving the estimates of a and b are given
the accompanying table.

Drawing on the information in the accompanying table we find that

110
Xi = 19.81 Yi= 391 (Xi-X)2 Yi-Y)2 Xi-X)(Yi-Y)

X = 0.7924 Y = 15.64 = 0.8311 =1661.76 = 10.007

Account Xi Yi (Xi-X) (Yi-Y) (Xi-X)2 (Yi-Y)2 (Xi-X)(Yi-Y)


Number

1 0.90 15 0.1076 -0.64 0.0116 0.4096 -0.0689


2 0.75 20 -0.0424 4.36 0.0018 19.0096 -0.1849
3 1.05 10 -0.2576 -5.64 0.0664 31.8096 -1.4529
4 0.85 14 0.0576 -1.64 0.0033 2.6896 -0.0945
G 5 0.65 16 -0.1424 0.36 0.0203 0.1296 -0.513
R 6 1.20 20 0.4076 4.36 0.1661 19.0096 1.7771
O 7 0.90 24 0.1076 8.36 0.0116 69.8896 0.8995
U 8 0.84 26 0.0476 10.36 0.0023 107.3296 0.4931
P 9 0.93 11 0.1376 -4.64 0.0189 21.5296 -0.6385
10 0.78 18 -0.0124 2.36 0.0002 5.5696 -0.0293
I 11 0.96 12 0.1676 -3.64 0.0281 13.2496 -0.6101
12 1.02 25 0.2276 9.36 0.0518 87.6096 2.1303
13 0.81 26 0.0176 10.36 0.0003 107.3296 0.1823
14 0.76 30 -0.0324 14.36 0.0010 206.2096 -0.4653
15 1.02 28 0.2276 12.36 0.0518 152.7696 2.8131

16 0.76 10 -0.0324 -5.64 0.0010 31.8069 0.1827


17 0.68 12 -0.1124 -3.64 0.0126 13.2496 0.4091
G 18 0.56 4 -0.2324 -11.64 0.0540 135.4896 2.7051
R 19 0.62 18 -0.1724 2.36 0.0297 5.5696 -0.4069
O 20 0.92 -4 0.1276 -19.64 0.0163 385.7296 -2.5061
U 21 0.58 20 -0.2124 4.36 0.0451 19.0096 -0.9261
P 22 0.70 8 -0.0924 - 7.64 0.0085 58.3696 0.7059
23 0.52 15 0.2724 -0.64 0.0742 0.4096 0.1743
II 24 0.45 6 0.3424 -9.64 0.1172 92.9296 3.3007
25 0.60 7 0.1924 -8.64 0.0370 74.6496 1.6623

19.81 391 0.8311 1661.76 9.539

Sum of Xi for group 1 13.42


X1 = = = 0.8947
15 15

Sum of Xi for group 2 6.39


X2 = = = 0.6390

111
10 10

Sum of Yi for group 1 295


Y1 = = = 19.67
15 15

Sum of Yi for group 2 96


Y2 = = = 9.60
10 10

1 0.8311
x 2 = Xi X)2 = = 0.0346
n-1 25-1

1 1661.76
y =
2
Yi Y) =
2
= 69.24
n-1 25-1

1 10.0007
xy = Xi-X)(Yi-Y) = = 0.4167
n-1 25-1

dx = X1 - X2 = 0.8947 0.6390 = 0.2557

dy = Y1 Y2 = 19.67 9.60 = 10.07

Substituting these values in the equations for a and b we get :

69.24 x 0.2557 0.4167 x 10.07


a = = 6.079
0.0346 x 69.24 0.4167 x 0.4167

0.0346 x 10.07 0.4167 x 0.2557


b= = 0.1089
0.0346 x 69.24 0.4167 x 0.4167

Hence , the discriminant function is :


Zi = 6.079 Xi + 0.1089 Yi

(b) Choice of the cutoff point


The Zi score for various accounts are shown below

Zi scores for various accounts

112
Account No. Zi Score

1 7.1046
2 6.7373
3 7.4720
4 6.6918
5 5.6938
6 9.4728
7 8.0847
8 7.9378
9 6.8514
10 6.7018
11 7.1426
12 8.9231
13 7.7554
14 7.8870
15 9.2498
16 5.7090
17 5.4405
18 3.8398
19 5.7292
20 5.1571
21 5.7038
22 5.1265
23 4.7946
24 3.3890
25 4.4097

The Zi scores arranged in an ascending order are shown below

Good(G)
Account Number Zi Score or
Bad (B)

24 3.3890 B
18 3.8398 B
25 4.4097 B
23 4.7946 B
22 5.1265 B
20 5.1571 B
17 5.4405 B
5 5.6938 G
21 5.7038 B

113
16 5.7090 B
19 5.7292 B
4 6.6918 G
10 6.7018 G
2 6.7373 G
9 6.8514 G
1 7.1046 G
11 7.1426 G
3 7.4720 G
13 7.7554 G
14 7.8870 G
8 7.9378 G
7 8.0847 G
12 8.9231 G
15 9.2498 G
6 9.4728 G

From the above table, it is evident that a Zi score which represents the mid-point between the
Zi scores of account numbers 19 and 4 results in the minimum number of misclassifications . This Zi
score is :

5.7292 + 6.6918
= 6.2105
2
Given this cut-off Zi score, there is just one misclassification (Account number 5)

114
Chapter 4
ANALYSING FINANCIAL PERFORMANCE

Net profit
1. Return on equity =
Equity

= Net profit Net sales Total assets


x x
Net sales Total assets Equity

1
= 0.05 x 1.5 x = 0.25 or 25 per cent
0.3

Debt Equity
Note : = 0.7 So = 1-0.7 = 0.3
Total assets Total assets

Hence Total assets/Equity = 1/0.3

2. PBT = Rs.40 million


PBIT
Times interest covered = = 6
Interest

So PBIT = 6 x Interest

115
PBIT Interest = PBT = Rs.40 million
6 x Interest = Rs.40 million
Hence Interest = Rs.8 million

3. Sales = Rs.7,000,000
Net profit margin = 6 per cent
Net profit = Rs.7000000 x 0.06 = 420,000
Tax rate = 60 per cent
420,000
So, Profit before tax = = Rs.1,050,000
(1-.6)
Interest charge = Rs.150,000

So Profit before interest and taxes = Rs.1,200,000


Hence

1,200,000
Times interest covered ratio = = 8
150,000

4. CA = 1500 CL = 600
Let BB stand for bank borrowing
CA+BB
= 1.5
CL+BB

1500+BB
= 1.5
600+BB

BB = 120

1,000,000
5. Average daily credit sales = = 2740
365
160000
ACP = = 58.4
2740

If the accounts receivable has to be reduced to 120,000 the ACP must be:
120,000
x 58.4 = 43.8days
160,000

116
Current assets
6. Current ratio = = 1.5
Current liabilities

Current assets - Inventories


Acid-test ratio = = 1.2
Current liabilities

Current liabilities = 800,000


Sales
Inventory turnover ratio = = 5
Inventories
Current assets - Inventories
Acid-test ratio = = 1.2
Current liabilities

Current assets Inventories


This means - = 1.2
Current liabilities Current liabilities

Inventories
1.5 - = 1.2
800,000

Inventories
= 0.3
800,000

Inventories = 240,000

Sales
=5 So Sales = 1,200,000
2,40,000

7. Debt/equity = 0.60
Equity = 50,000 + 60,000 = 110,000
So Debt = 0.6 x 110,000 = 66,000
Hence Total assets = 110,000+66,000 = 176,000
Total assets turnover ratio = 1.5
So Sales = 1.5 x 176,000 = 264,000
Gross profit margin = 20 per cent

117
So Cost of goods sold = 0.8 x 264,000 = 211,200
Days sales outstanding in accounts receivable = 40 days
Sales
So Accounts receivable = x 40
360

264,000
= x 40 = 29,333
360

Cost of goods sold 211,200


Inventory turnover ratio = = = 5
Inventory Inventory

So Inventory = 42,240

Assuming that the debt of 66,000 represent current liabilities


Cash + Accounts receivable
Acid-test ratio =
Current liabilities

Cash + 29,333
= = 1.2
66,000
So Cash = 49867

Plant and equipment = Total assets - Inventories Accounts receivable Cash


= 176,000 - 42240 - 29333 49867
= 54560

Pricing together everything we get

Balance Sheet
Equity capital 50,000 Plant & equipment 54,560
Retained earnings 60,000 Inventories 42,240
Debt(Current liabilities) 66,000 Accounts receivable 29,333
Cash 49,867

176,000 176,000

Sales 264,000
Cost of goods sold 211,200

Cash & bank balances + Receivables + Inventories + Pre-paid expenses

118
8. (i) Current ratio =
Short-term bank borrowings + Trade creditors + Provisions

5,000,000+15,000,000+20,000,000+2,500,000
=
15,000,000+10,000,000+5,000,000

42,500,000
= = 1.42
30,000,000

Current assets Inventories 22,500,000


(ii) Acid-test ratio = = = 0.75
Current liabilities 30,000,000

Long-term debt + Current liabilities


(iii) Debt-equity ratio =
Equity capital + Reserves & surplus

12,500,000 + 30,000,000
= = 1.31
10,000,000 + 22,500,000

Profit before interest and tax


(iv) Times interest coverage ratio =
Interest

15,100,000
= = 3.02
5,000,000

Cost of goods sold 72,000,000


(v) Inventory turnover period = = = 3.6
Inventory 20,000,000
365
(vi) Average collection period =
Net sales/Accounts receivable
365
= = 57.6 days
95,000,000/15,000,000

Net sales 95,000,000

119
(vii) Total assets turnover ratio = = = 1.27
Total assets 75,000,000

Profit after tax 5,100,000


(ix) Net profit margin = = = 5.4%
Net sales 95,000,000

PBIT 15,100,000
(x) Earning power = = = 20.1%
Total assets 75,000,000

Equity earning 5,100,000


(xi) Return on equity = = = 15.7%
Net worth 32,500,000

The comparison of the Omexs ratios with the standard is given below

Omex Standard
Current ratio 1.42 1.5
Acid-test ratio 0.75 0.80
Debt-equity ratio 1.31 1.5
Times interest covered ratio 3.02 3.5
Inventory turnover ratio 3.6 4.0
Average collection period 57.6 days 60 days
Total assets turnover ratio 1.27 1.0
Net profit margin ratio 5.4% 6%
Earning power 20.1% 18%
Return on equity 15.7% 15%

Note that solutions to problems 10 & 11 are not given

MINICASE

Solution:

(a) Key ratios for 20 X 5


12.4
Current ratio = = 0.93
13.4

8.8 + 6.7
Debt-equity ratio = = 0.98

120
6.5 + 9.3

57.4
Total assets turnover ratio = = 1.96
[(34 6.6) + (38 6.7)] / 2

3.0
Net profit margin = = 5.2 percent
57.4

5
Earning power = = 17.0 percent
[(34 6.6) + (38 6.7)] / 2

3.0
Return on equity = = 20.2 percent
(13.9 + 15.8) / 2

(b) Dupont Chart for 20 x 5

Net sales +/-


Non-op. surplus
deficit 57.8

Net profit
3.0

Net profit Total costs


margin 54.8
5.2%

Net sales
57.4
Return on
total assets
10.2%

Net sales
57.4

Total asset Average


turnover fixed assets
1.96 21.4

121
+
Average total
assets Average
29.35 net current
assets 54.0

Average
other assets
2.55

(c) Common size and common base financial statements

Common Size Financial Statements


Profit and Loss Account

Regular (in million) Common Size (%)


20 X 4 20 X 5 20 X 4 20 X 5
Net sales 39.0 57.4 100 100
Cost of goods sold 30.5 45.8 78 80
Gross profit 8.5 11.6 22 20
Operating expenses 4.9 7.0 13 12
Operating profit 3.6 4.6 9 8
Non-operating surplus / 0.5 0.4 1 1
deficit
PBIT 4.1 5.0 11 9
Interest 1.5 2.0 4 3
PBT 2.6 3.0 7 5
Tax - - - -
Profit after tax 2.6 3.0 7 5

Balance Sheet

Regular (in million) Common Size (%)


20 X 4 20 X 5 20 X 4 20 X 5

122
Shareholders funds 13.9 15.8 51 50
Loan funds 13.5 15.5 49 50
Total 27.4 31.3 100 100
Net fixed assets 19.6 23.2 72 74
Net current assets 5.1 5.7 19 18
Other assets 2.7 2.4 10 8
Total 27.4 31.3 100 100

Common Base Year Financial Statements


Profit and Loss Account

Regular (in million) Common Base Year(%)


20 X 4 20 X 5 20 X 4 20 X 5
Net sales 39.0 57.4 100 147
Cost of goods sold 30.5 45.8 100 150
Gross profit 8.5 11.6 100 136
Operating expenses 4.9 7.0 100 43
Operating profit 3.6 4.6 100 128
Non-operating surplus / 0.5 0.4 100 80
deficit
PBIT 4.1 5.0 100 122
Interest 1.5 2.0 100 133
PBT 2.6 3.0 100 115
Tax - - 100 100
Profit after tax 2.6 3.0 100 115

Balance Sheet

Regular (in million) Common Base Year(%)


20 X 4 20 X 5 20 X 4 20 X 5
Shareholders funds 13.9 15.8 100 114
Loan funds 13.5 15.5 100 115
Total 27.4 31.3 100 114

123
Net fixed assets 19.6 23.2 100 118
Net current assets 5.1 5.7 100 112
Other assets 2.7 2.4 100 89
Total 27.4 31.3 100 114

(d) The financial strengths of the company are:

Asset productivity appears to be good.


Earning power and return on equity are quite satisfactory
Revenues have grown impressively over 20 x 4 20 x 5

The financial weaknesses of the company are:

Current ratio is unusually low


While revenues grew impressively, costs rose even faster: As a result profit margins
declined
The company did not have any tax liability in the last two years. If the company has to
bear the burden of regular taxes, its return on equity will be adversely impacted

(e) The following are the problems in financial statement analysis

There is no underlying theory


It is difficult to find suitable benchmarks for conglomerate firms
Firms may resort to window dressing
Financial statements do not reflect price level changes
Diversity of accounting policies may vitiate financial statement analysis
It is somewhat difficult to judge whether a certain ratio is good or bad

(f) The qualitative factors relevant for evaluating the performance and prospects of a
company are as follows:

Are the companys revenues tied to one key customer?


To what extent are the companys revenues tied to one key product?
To what extent does the company rely on a single supplier?
What percentage of the companys business is generated overseas?
How will competition impact the company?
What are the future prospects of the firm?
What could be the effect of the changes in the legal and regulatory environment?

124
Chapter 5
BREAK-EVEN ANALYSIS AND LEVERAGES

1. a. EBIT = Q(P-V)-F
= 20,000(10-6)-50,000 = Rs.30,000

b. EBIT = 12,000(50-30)-200,000 = Rs.40,000

2. EBIT = Q(P-V)-F
EBIT=Rs.30,000 , Q=5,000 , P=Rs.30 , V=Rs.20
So, 30,000 = 5,000(30-20)-F
So, F = Rs.20,000.

Q(P-V)
3. DOL =
Q(P-V)-F

P=Rs.1,000 ,V=Rs.600, F=Rs.100,000

400(1,000-600)
DOL(Q=400) = = 2.67
400(1,000-600)-100,000

600(1,000-600)
DOL(Q=600) = = 1.71

125
600(1,000-600)-1,00,000

4. DOL(Q=15000) = 2.5
EBIT(Q=15000) = Rs.3,00,000

Percentage change in EBIT = DOL x Percentage change in Q


If the percentage change in Q is 10%
Percentage change in EBIT = 2.5 x 10% = - 25%
If the percentage change in Q is + 5%
Percentage change in EBIT = 2.5 x 5% = 12.5%

Hence the possible forecast errors of EBIT in percentage terms is 25% to


12.5%
The corresponding value range of EBIT is Rs.225,000 to Rs.337,500

5. Break even point in units


F 50,000
Q = = =10,000 units
P-V 12-7

Break even point in rupees:


Q x P = 10,000 x Rs.12 = Rs,120,000
To earn a pre-tax income of Rs.60,000 the number of units to be sold is

F + Target pre-tax income


Q =
P-V
= 50,000 + 60,000
= 22,000 units
12-7
To earn an after-tax income of Rs.60,000 if the tax rate is 40 per cent, the
Pre-tax income must be Rs.60,000
= Rs.100,000
1-.4
Hence the number of units to be sold to earn an after-tax income of Rs.60,000
is :
50,000 + 100,000
Q = = 30,000 units
12-7

6. P-V
= 0.30 P-V = Rs.6 F=20,000

126
P

20000 6
Q = = 3,333 P = = Rs.20
6 0.30

Break even point in rupees = Rs.66,666

When net income is Rs.60,000


20,000 +60,000
Q = = 13,333
6
Sales in rupees = 13,333 x Rs.20 = Rs.266,666

10,000
7. (a) P = Rs.30 ,V=Rs.16, F=Rs.10,000 Q = = 714.3 bags
30-16

(b) Profit when the quantity is 3000 bags


Profit =3,000(30-16)-10000 = Rs.32000
10 per cent increase in production means that the quantity is 3300 bags
At that production
Profit = 3,300(30-16)-10,000 = Rs.36200
So, the percentage change in profit is :
36200-32000
= 13.1%
32000

(c) A 10 per cent increase in selling price means that P= Rs.33


Break-even point when P= Rs.33

10,000
Q = = 588.2 bags
33-16

(d) A 50 per cent increase in fixed costs means that F=Rs.15,000


Break-even point when F= Rs.15,000
15,000
Q = = 882.4 bags
33-16
(e) If V= Rs.20, the break-even point is :

127
10,000
Q = = 1000 bags
30-20

8. A B C D
Selling price per unit Rs.10 Rs.16.66 Rs.20 Rs.10
Variable cost per unit Rs.6 Rs.8.33 Rs.12 Rs.5
Contribution margin per unit Rs.4 Rs.8.33 Rs.8 Rs.5
Contribution margin ratio 0.4 0.5 0.4 0.5
Total fixed costs Rs.16000 Rs.100000 Rs.160000 Rs.60000
Break-even point in units 4000 12000 20000 12000
Break-even sales(Rs.) Rs.40000 Rs.200000 Rs.400000 Rs.120000
Net income(loss)before tax Rs.30000 Rs.80000 Rs.(40000) Rs.40000
No.of units sold 11500 21600 15000 20000

9. (a) Break-even point for product P


30,000
= 3,000 units
30-20

Break-even point for product Q


100,000
= 5,000 units
50-30
Break-even point for product R
200,000
= 5,000 units
80-40

(b) The weighted contribution margin is :


5000 8,000 6,000
x Rs.10 + x Rs.20 + x Rs.40 = Rs.23.68
19000 19000 19000

10. EBIT
DFL =
Dp
EBIT I -
T
at Q = 20000

128
EBIT= 20000(Rs.40-Rs.24)=Rs.320,000

Rs.320,000
DFL(Q=20,000) =
Rs.10,000
Rs.320,000-Rs.30,000 -
(1-.5)
= 1.185

11. (a) EBIT = Q(P-V) F

Firm A : 20,000(Rs.20-Rs.15) Rs.40,000 = Rs.60,000


Firm B : 10,000(Rs.50-Rs.30) - Rs.70,000 = Rs.130,000
Firm C : 3,000(Rs.100-Rs.40)- Rs.100,000 = Rs.80,000
(EBIT-I) (1-T) - Dp
(b) EPS =
n

(Rs.60,000-Rs.10,000)(1-.4)-Rs.5,000
Firm A : = Rs.1.9
10,000

(Rs.130,000-Rs.20,000)(1-.5)-Rs.5,000
Firm B : = Rs.4.17
12,000

(Rs.80,000-Rs.40,000)(1-.6)-Rs.10,000
Firm C : = Rs.0.40
15,000
F+I
(c) BEP =
PV

Rs.40,000 + Rs.10,000
Firm A : = 10,000 units
Rs.20 Rs.15

Rs.70,000 + Rs.20,000
Firm B : = 4,500 units
Rs.50 Rs.30

Rs.100,000 + Rs.40,000
Firm C : = 2,333 units

129
Rs.100 Rs.40

Q(P-V)
(d) DOL =
Q(P-V)-F

20,000(Rs.20-Rs.15)
Firm A : = 1.67
20,000(Rs.20-Rs.15)- Rs.40,000

10,000(Rs.50-Rs.30)
Firm B : = 1.54
10,000(Rs.50-Rs.30)-Rs.70,000

3,000(Rs.100-Rs.40)
Firm C : = 2.25
3,000(Rs.100-Rs.40)-Rs.100,000

EBIT
(e) DFL =
Dp
EBIT I -
(1-T)

Rs.60,000
Firm A : = 1.44
Rs.5000
Rs.60,000-Rs.10,000 -
(1-.4)

Rs.130,000
Firm B : = 1.30
Rs.5,000
Rs.130,000-Rs.20,000 -
(1-.5)

Rs.80,000
Firm C : = 5.333
Rs.10,000
Rs.80,000-Rs.40,000-
(1-.6)
(f) DTL = DOL x DFL

Firm A : 1.67 x 1.44 = 2.40

130
Firm B : 1.54 x 1.30 = 2.00
Firm C : 2.25 x 5.333 = 12.00

Chapter 6
FINANCIAL PLANNING AND BUDGETING

1. The proforma income statement of Modern Electronics Ltd for year 3 based on the per cent
of sales method is given below

Average per cent Proforma income statement


of sales for year 3 assuming sales of
1020

Net sales 100.0 1020.0


Cost of goods sold 76.33 778.57
Gross profit 23.67 241.43
Selling expenses 7.40 75.48
General & administration expenses 6.63 67.63
Depreciation 6.75 68.85
Operating profit 2.90 29.58
Non-operating surplus/deficit 1.07 10.91
Earnings before interest and taxes 3.96 40.39
Interest 1.24 12.65
Earnings before tax 2.72 27.74
Tax 1.00 10.20
Earnings after tax 1.72 17.54
Dividends (given) 8.00
Retained earnings 9.54

131
2. The proforma income statement of Modern Electronics for year 3 using the
the combination method is given below :

Average per cent Proforma income statement


of sales for year 3

Net sales 100.0 1020.0


Cost of goods sold 76.33 778.57
Gross profit 23.67 241.43
Selling expenses 7.40 75.48
General & administration expenses Budgeted 55.00
Depreciation Budgeted 60.00
Operating profit 50.95
Non-operating surplus/deficit 1.07 10.91
Earnings before interest and taxes 61.86
Interest Budgeted 12.0
Earnings before tax 49.86
Tax 1.00 10.20
Earnings after tax 39.66
Dividends (given) Budgeted 8.00
Retained earnings 31.66

132
3. The proforma balance sheet of Modern Electronics Ltd for year 3 is given below

Average of percent Projections for year 3


of sales or some based on a forecast
other basis sales of 1400

Net sales 100.0 1020.0

ASSETS
Fixed assets (net) 40.23 410.35
Investments No change 20.00

Current assets, loans & advances :


Cash and bank 1.54 15.71
Receivables 22.49 229.40
Inventories 21.60 220.32

Prepaid expenses 5.09 51.92


Miscellaneous expenditure & losses No change 14.00

961.70

LIABILITIES :

Share capital :
Equity No change 150.00
Reserves & surplus Proforma income 160.66
statement

Secured loans:

133
Term loans No change 175.00
Bank borrowings No change 199.00

Current liabilities :
Trade creditors 17.33 176.77
Provisions 5.03 51.31

External funds requirement Balancing figure 48.96

961.7

A L
4. EFR = - S m S1 (1-d)
S S

800 190
= - 300 0.06 x 1,300 (1-0.5)
1000 1000

= (0.61 x 300) (0.06) x 1,300 x (0.5)

= 183 39 = Rs.144.

Projected Income Statement for Year Ending 31st December , 2001

Sales 1,300
Profits before tax 195
Taxes 117
Profit after tax (6% on sales) 78
Dividends 39
Retained earnings 39

Projected Balance Sheet as at 31.12 2001

Liabilities Assets

Share capital 150 Fixed assets 520


Retained earnings 219 Inventories 260
Term loans (80+72) 152 Receivables 195

134
Short-term bank borrowings 272 Cash 65
(200 + 72)
Accounts payable 182
Provisions 65

1,040 1,040

A L
5. (a) EFR = - S m S1 (1 d)
S S

150 30
= - x 80 (0.625) x 240 x (0.5)
160 160

= (60 7.5) = 52.5

(b) Projected Balance Sheet as on 31.12.20X1

Liabilities Assets

Share capital 56.25 Net fixed assets 90


Retained earnings 47.50 Inventories 75
(40 + 7.5)
Term loans 46.25 Debtors 45
Short-term bank 30.00 Cash 15
borrowings
Trade creditors 37.50
Provisions 7.50

225.00 225.00

(c) 20X0 20X1


i) Current ratio 1.50 1.80
ii) Debt to total assets ratio 0.53 0.54
iii) Return on equity 14.3% 14.5%

(d)
A L
EFR 20X1= - S mS1 (1 d)

135
S S

150 30
= - 20 0.0625 x 180 x 0.5
160 160

= 9.38

150 x (1.125) 30 x 1.125


EFR 20X2 = - x 20 0.0625 x 200 x 0.5
180 180

168.75 33.75
= - x 20 0.0625 x 220 x 0.5
180 180

= 8.75

168.75 x (1.11) 33.75 x (1.11)


EFR 20X3 = - 20 0.0625 x 220 x 0.5
200 200

187.31 37.46
= - x 20 6.88
200 200

= 8.11

187.31 x (1.1) 37.46 x (1.1)


EFR 20X4 = - x 20 0.0625 x 240 x 0.5
220 220

= 7.49

Balance Sheet as on 31st December, 20X4

Liabilities Rs. Assets Rs.

Share capital 46.87 Net fixed assets 90.00


(30+16.87) (60 x 240/160)

136
Retained earnings Inventories
(40.00+5.63+6.25+6.88+7.50) 66.26 (50x240/160) 75.00
Term loans(20+16.87) 36.87 Debtors (30x240/160) 45.00
Short-term bank borrowings 30.00 Cash (10x240/160) 15.00
Trade creditors 37.50
Provisions 7.50

225.00 225.00

6. EFR A L m (1+g) (1-d)


= - -
S S S g
Given A/S= 0.8 , L/S= 0.5 , m= 0.05 , d= 0.6 and EFR = 0 we have,

(0.05)(1+g)(0.4)
(0.8-0.5) - =0
g

(0.05)(1+g)(0.4)
i.e. 0.3 - =0
g

Solving the above equation we get g = 7.14%

A L
7. (a) EFR = - S mS1 (1-d)
S S

320 70
= - x 100 (0.05) (500) (0.5)
400 400

= Rs.50

(b) Let CA = denote Current assets


CL = Current liabilities
SCL = Spontaneous current liabilities
STL = Short-term bank borrowings
FA = Fixed assets
and LTL = Long-term loans

i. Current ratio

137
CA
i.e greater than or equal to 1.25 or
CL

CA

STL +SCL

As at the end of 20X1, CA = 20x0 x 1.25 = 237.50


SCL = 70 x 1.25 = 87.50
Substituting these values, we get
1.25 (STL + 87.5) 237.50
or 1.25 STL x


or STL =
1.25
i.e STL Rs.102.50

ii. Ratio of fixed assets to long term loans 1.25


FA

LTL
At the end of 20X1 FA = 130 x 1.25 = 162.5
162.5
LTL or LTL = Rs.130
1.25
If STL and LTL denote the maximum increase in ST borrowings & LT

borrowings , we have :
STL = STL (20X1) STL (20X1) = 102.50 60.00 = 42.50
LTL = LTL (20X1)- LTL (20X1) = 130.00 80.00 = 50.00
Hence, the suggested mix for raising external funds will be :
Short-term borrowings 42.50
Long-term loans 7.50
Additional equity issue --

50.00

A L
8. EFR = - S m S1 (1-d)
S S

138
A S
Therefore, mS1(1-d) - S represents surplus funds
S S
Given m= 0.06, S1 =11,000, d= 0.6 , L= 3,000 S= 10,000 and
surplus funds = 150 we have
A 3,000
(0.06) 11,000 (1-0.6) - - 1,000 = 150
10,000 10,000

A 3,000
= (0.06) (0.4) (11,000) 150 = 114
10

or A = (1,140 + 3,000) = 4,140

The total assets of Videosonics must be 4,140

9. m= .05 , d = 0.6 , A/E = 2.5 , A/S = 1.4

m (1-d)A/E .05 (1-0.6) 2.5


(a) g= = = 3.70 per cent
A/S m(1-d)A/E 1.4 -.05 (1-0.6) 2.5

.05 (1-0.6) x A/E


(b) 0.5 = A/E = 3.33
2.4 - .05 (1-0.6) A/E

d = 0.466
The dividend payout ratio must be reduced from 60 per cent to 46.6 per cent

.05 (1-0.6) x A/E


(c) .05 = A/E = 3.33
1.4 -.05 (1-0.6) A/E

The A/E ratio must increase from 2.5 to 3.33

m (1-0.6) 2.5
(d) .06 = m = 7.92 per cent
1.4 m (1-0.6) x 2.5

The net profit margin must increase from 5 per cent to 7.92 per cent

.05 (1-0.6) 2.5


(e) .06 = A/S = .883

139
A/S - .05 (1-0.6) 2.5

The asset to sales ratio must decrease from 1.4 to 0.883

Chapter 32
CORPORATE VALUATION

1. (a) The calculations for Hitech Limited are shown below :


Year 2 Year3
EBIT
PBT 86 102
+ Interest expense 24 28
- Interest income (10) (15)
- Non-operating income (5) (10)
EBIT 95 105

Tax on EBIT
Tax provision on income statement 26 32
+ Tax shield on interest expense 9.6 11.2
- Tax on interest income (4) (6)
- Tax on non-operating income (2) (4)
Tax on EBIT 29.6 33.2

NOPLAT 65.4 71.8


Net investment (50) (50)
Non-operating cash flow (post-tax) 3 6
FCFF 18.4 27.8

(b) The financing flow for years 2 and 3 is as follows :


Year 2 Year 3
After-tax interest expense 14.4 16.8
Cash dividend 30 40
- Net borrowings (30) (30)

140
+ Excess marketable securities 30 10
- After-tax income on excess (6) (9)
marketable securities
- Share issue (20) -
18.4 27.8

(c) Year 2 Year 3


Invested capital (Beginning) 310 360
Invested capital (Ending) 360 410
NOPLAT 65.4 71.8
Turnover 400 460
Net investment 50 50

Post-tax operating margin 16.35% 15.61%


Capital turnover 1.29 1.28
ROIC 21.1% 19.9%
Growth rate 16.1% 13.9%
FCF 15.4 21.8

2. Televista Corporation

0 1 2 3 4 5
Base year

1. Revenues 1600 1920 2304 2765 3318 3650


2. EBIT 240 288 346 415 498 547
3. EBIT (1-t) 156 187 225 270 323 356
4. Cap. exp. 200 240 288 346 415 -
- Depreciation 120 144 173 207 249
5. Working capital 400 480 576 691 829 912
6. Working capital 80 96 115 138 83
7. FCFF 11 13 16 19 273
(3-4-6)

Discount factor 0.876 0.767 0.672 .589


Present value 9.64 9.97 10.76 11.19

Cost of capital for the high growth period

0.4 [12% + 1.25 x 7%] + 0.6 [15% (1 - .35)]


8.3% + 5.85%
= 14.15%

141
Cost of capital for the stable growth period
0.5 [12% + 1.00 x 6%] + 0.5 [14% (1 - .35)]
9% + 4.55%
= 13.55%

Present value of FCFF during the explicit forecast period


= 9.64 + 9.97 + 10.76 + 11.19 = 41.56

273 273
Horizon value = = = 7690
0.1355 0.10 0.0355

Present value of horizon value = 4529.5

Value of the firm = 41.56 + 4529.50 = Rs.4571.06 million

3. The WACC for different periods may be calculated :

WACC in the high growth period

Year kd(1-t) = 15% (1-t) ke = Rf + x Market risk premium ka = wd kd (1-t)+ we ke


1 15 (0.94) = 14.1% 12 + 1.3 x 7 = 21.1% 0.5 x 14.1 + 0.5 x 21.1 = 17.6%
2 15 (0.88) = 13.2% 21.1% 0.5 x 13.2 + 0.5 x 21.1 = 17.2%
3 15 (0.82) = 12.3% 21.1% 0.5 x 12.3 + 0.5 x 21.1 = 16.7%
4 15 (0.76) = 11.4% 21.1% 0.5 x 11.4 + 0.5 x 21.1 = 16.3%
5 15 (0.70) = 10.5% 21.1% 0.5 x 10.5 + 0.5 x 21.1 = 15.8%

WACC in the transition period


kd(1-t) = 14 (1 0.3) = 9.8%
ke = 11 + 1.1 x 6 = 17.6%
ka = 0.44 x 9.8 + 0.56 x 17.6 = 14.2%

WACC for the stable growth period


kd(1-t) = 13 (1 0.3) = 9.1%
ke = 11 + 1.0 x 5 = 16%
ka = 1/3 x 9.1 + 2/3 x 16 = 13.7%

The FCFF for years 1 to 11 is calculated below. The present value of the
FCFF for the years 1 to 10 is also calculated below.

Multisoft Limited

142
Period Growth EBIT Tax EBIT Cap. Dep. WC FCFF D/E Beta WACC PV Present
rate (%) rate (1-t) exp. % Factor value
(%)
0 90 100 60
1 40 126 6 118 140 84 26 36 1:1 1.3 17.6 .850 30.6
2 40 176 12 155 196 118 39 38 1:1 1.3 17.2 .726 27.6
3 40 247 18 203 274 165 50 44 1:1 1.3 16.7 .622 27.4
4 40 346 24 263 384 230 70 39 1:1 1.3 16.3 .535 20.8
5 40 484 301 339 2 3
538 323 98 4 26 5 1.3 15.8
1:1 6 .462 12.0
61. Revenues
34 649 950 4541,000 721 1,200
30 432 1321,450 33 1,660
0.8:1 1.1 1,770
14.2 .405 13.4
72. PBIT 28 830 140
30 581 115 130
922 553 169 22243 245 287
0.8:1 1.1 14.2 .354 15.4
83. NOPAT 22 = PBIT1013 30 91 709 74.8 1125 675 84.5 206 144.3 53 159.31.1 14.2
0.8:1 186.6 .310 16.7
9 (1 .35)
16 1175 30 822 1305 783 239 61 0.8:1 1.1 14.2 .272 16.9
4.
10 Depreciation
10 1292 30 55 905 85 80
1436 862 263 8368 85 1.1 14.2
0.8:1 87 .238 16.6
5.
11 Gross cash
10 flow1421 30146 995 159.8 164.5
1580 948 289 227.3
74 244.3 273.7
0.5: 1.1 13.7 476
6. Gross investment 100 250 85 100 105
1.0 120
in fixed assets 673.4
7. Investment in net 10 value of15continuing70value is : 70
The present 70 54
current assets
8. Total investment
FCF11 110 265 15574 170 175 174
9. FCFF (5) (8) x PV factor36 10 years
(105.2)= 9.5 57.3
x 0.238 = 69.3
476 99.6
kg 0.137 0.100
0.4 1.0
WACC This is shown in the present value cell against{8
= x 12 x (1 0.35) + + 1.06
year 11. (8)}
1.4 1.4
The value of the firm is equal to :
= Present
14% value of FCFF during + Present value of continuing
The explicit forecast period of 10 years value
99.6 (1.10)
ContinuingThis
Value = up to Rs.685.4
adds = 2739.00
million as shown below
0.14 0.10

2739
MINI CASE
Present value of continuing value = = 1249
6
Solution: (1.14)
Solution:

PV of the FCFF during the explicit forecast period


3.6 105.2 9.5 57.3 69.3 99.6
= + + + +
(1.14) (1.14)2 (1.14)3 (1.14)4 (1.14)5 (1.14)6

= 72.4
Firm value = 72.4 + 1249 = 1321.4
143
Value of equity = 1321.4 200 = 1121.4 million
Chapter 33
VALUE BASED MANAGEMENT

1. The value created by the new strategy is calculated below :

Current Income Statement Projection


Values
(Year 0) 1 2 3 4 5

Sales 2000 2240 2509 2810 3147 3147


Gross margin (20%) 400 448 502 562 629 629

144
Selling and general 160 179 201 225 252 252
administration (8%)
Profit before tax 240 269 301 337 378 378
Tax 72 81 90 101 113 113
Profit after tax 168 188 211 236 264 264

Balance Sheet Projections


Fixed assets 600 672 753 843 944 944
Current assets 600 672 753 843 944 944
Total assets 1200 1344 1505 1696 1888 1888
Equity 1200 1344 1505 1686 1888 1888

Cash Flow Projections


Profit after tax 188 211 236 264 264
Depreciation 60 67 75 84 94
Capital expenditure 132 148 166 185 94
Increase in current assets 72 81 90 101 -
Operating cash flow 44 49 55 62 264

Present value of the operating cash flow = 147


Residual value = 264 / 0.15 = 1760
Present value of residual value = 1760 / (1.15)4 = 1007
Total shareholder value = 147 + 1007 = 1154
Pre-strategy value = 168/0.15 = 1120
Value of the strategy = 1154 1120 = 34

2. According to the Marakon approach


M rg
=
B kg

r - .10
2 =
k - .10
r - .10 = 2k - .20
r = 2k - .10
r/k = 2 - (.10/k)

Thus r/k is a function of k. Unless k is specified r/k cannot be determined.

3. (a) NOPAT for 20X1


PBIT (1 T) = 24 (0.65) = 15.6

145
(b) Return on capital for 20X1
NOPAT 15.6
= = 15.6%
Capital employed 120 20 (Non-interest bearing liabilities)

(c) Cost of equity


6% + 0.9 (6%) = 1.4%

(d) Average cost of capital


0.5 x 8% (1 - .35) + 0.5 x 11.4% = 8.3%

(e) EVA for 20X1


NOPAT - Average cost of capital x Capital employed
15.6 - .083 x 100 = 7.3

4.
I = Rs.200 million
r = 0.40
c* = 0.20
T = 5 years
200 (0.40 0.20) 5
Value of forward plan =
0.20 (1.20)

= Rs.833.3 million

5. Cost of capital = 0.5 x 0.10 + 0.5 x 0.18 = 0.14 or 14 per cent

1. Revenues 2,000 2,000 2,000 2,000 2,000


2. Costs 1,400 1,400 1,400 1,400 1,400
3. PBDIT 600 600 600 600 600
4. Depreciation 200 200 200 200 200
5. PBIT 400 400 400 400 400
6. NOPAT 240 240 240 240 240
7. Cash flow (4+6) 440 440 440 440 440
8. Capital at charge 1,000 800 600 400 200
9. Capital charge (8x0.14) 140 112 84 56 28
10. EVA (6-9) 100 128 156 184 212
5 440
NPV = - 1000 = 440 x 3.433 1000 = 510.5
t
t=1 (1.14)

EVAt
NPV = = 100 x 0.877 + 128 x 0.769 + 156 x 0.675 + 184 x 0.592 +

146
(1.14)t 212 x 0.519
= 510.3

6. Equipment cost = 1,000,000


Economic life = 4 years
Salvage value = Rs.200,000
Cost of capital = 14 per cent

Present value of salvage value = 200,000 x 0.592


= 118,400

Present value of the annuity = 1,000,000 118,400


= 881,600

881,600 881,600
Annuity amount = =
PVIFA14%, 4yrs 2.914

= Rs.302,540

Depreciation charge under sinking fund method


1 2 3 4
Capital 1,000,000 837,460 652,164 440,927
Depreciation 162,540 185,296 212,237 240,810
Capital charge 140,000 117,244 91,303 61,730
Sum 302,540 302,540 302,540 302,540

7. Investment : Rs.2,000,000
Life : 10 years
Cost of capital : 15 per cent
Salvage value : 0

2,000,000
Economic depreciation =
FVIFA(10yrs, 15%)

2,000,000
= = 98,503
20.304

8. Investment : Rs.5,000,000
Life : 5 years
Cost of capital : 12 per cent
Salvage value : Nil

147
PVIFA(5yrs,12%) = 3.605 ; Annuity amount = 5,000,000 / 3.605 = 1,386,963

Depreciation charge under sinking fund method


1 2 3 4 5
Capital 5,000,000 4,213,037 3,331,638 2,344,472 1,238,846
Depreciation 786,963 881,399 987,166 1,105,626 1,238,301
Capital charge 600,000 505,564 399,797 281,336 148,662
Sum 1,386,963 1,386,963 1,386,963 1,386,963 1,386,963

5,000,000
Economic depreciation =
FVIFA(5yrs, 12%)

5,000,000
= = Rs.787,030
6.353

9. Investment = Rs.100 million


Net working capital = Rs.20 million
Life = 8 yrs
Salvage value = Rs.20 million (Net working capital)
Annual cash flow = Rs.21.618 million
Cost of capital = 15%
Straight line depreciation = Rs.10 million per year

80 80
Economic depreciation = = = Rs.5.828 million
FVIFA(8, 15%) 13.727

Year 1 Year 4
Profit after tax 11.618 11.618
Depreciation 10.000 10.000
Cash flow 21.618 21.618
Book capital100 70
(Beginning)
ROCE 11.62% 16.59%
ROGI 21.62% 21.62%
CFROI 15.79% 15.79%

148
Chapter 34
MERGERS, ACQUISITIONS AND RESTRUCTURING

149
1. The pre-amalgamation balance sheets of Cox Company and Box Company and the post-
amalgamation balance sheet of the combined entity, Cox and Box Company, under the pooling
method as well as the purchase method are shown below :

Before Amalgamation After Amalgamation


Cox & Box Company
Cox Box Pooling method Purchase
method
Fixed assets 25 10 35 45
Current assets 20 7.5 27.5 30
Goodwill 2.5
Total assets 45 17.5 62.5 77.5

Share capital 20 5 25 20
(face value @ Rs.10)
Reserves & surplus 10 10 20 10
Share premium 15 2.5 17.5 17.5
Debt 45 17.5 42.5 77.5

2. Post-merger EPS of International Corporation will be

2 x 100,000 + 2 x100,000

100,000 + ER x 100,000

Setting this equal to Rs.2.5 and solving for ER gives


ER = 0.6

3. PVA = Rs.25 million, PVB = Rs.10 million


Benefit = Rs.4 million, Cash compensation = Rs.11 million
Cost = Cash compensation PVB = Rs.1 million
NPV to Alpha = Benefit Cost = Rs.3 million
NPV to Beta = Cash Compensation PVB = Rs.1 million

4. Let A stand for Ajeet and J for Jeet


PVA = Rs.60 x 300,000 = Rs.18 million
PVJ = Rs.25 x 200,000 = Rs.5 million
Benefit = Rs.4 million
PVAJ = 18 + 5 + 4 = Rs.23 million
Exchange ratio = 0.5
The share of Jeet in the combined entity will be :
100,000
= = 0.25

150
300,000 + 100,000

a) True cost to Ajeet Company for acquiring Jeet Company


Cost = PVAB - PVB
= 0.25 x 27 - 5 = Rs.1.75 million

b) NPV to Ajeet
= Benefit - Cost
= 4 - 1.75 = Rs.2.25 million

c) NPV to Jeet = Cost = Rs.1.75 million

5. a) PVB = Rs.12 x 2,000,000 = Rs.24 million


The required return on the equity of Unibex Company is the value of k in the
equation.

Rs.1.20 (1.05)
Rs.12 =
k - .05

k = 0.155 or 15.5 per cent.

If the growth rate of Unibex rises to 7 per cent as a sequel to merger, the intrinsic value
per share would become :

1.20 (1.07)
= Rs.15.11
0.155 - .07

Thus the value per share increases by Rs.3.11 Hence the benefit of the
acquisition is
2 million x Rs.3.11 = Rs.6.22 million

(b) (i) If Multibex pays Rs.15 per share cash compensation, the cost of the
merger is 2 million x (Rs.15 Rs.12) = Rs.6 million.

(ii) If Multibex offers 1 share for every 3 shares it has to issue 2/3 million
shares to shareholders of Unibex.

So shareholders of Unibex will end up with

0.667
= 0.1177 or 11.77 per cent
5+0.667

151
shareholding of the combined entity,
The present value of the combined entity will be
PVAB = PVA + PVB + Benefit
= Rs.225 million + Rs.24 million + Rs.6.2 million
= Rs.255.2 million

So the cost of the merger is :


Cost = PVAB - PVB
= .1177 x 255.2 - 24 = Rs.6.04 million

6. The expected profile of the combined entity A&B after the merger is shown in the last column
below.

A B A&B
Number of shares 5000 2000 6333
Aggregate earnings Rs.45000 Rs.4000 Rs.49000
Market value Rs.90000 Rs.24000 Rs.114000
P/E 2 6 2.33

7. (a) The maximum exchange ratio acceptable to shareholders of Vijay Limited is :

S1 (E1+E2) PE12
ER1 = - +
S2 P1S2

12 (36+12) 8
= - + = 0.1
8 30 x 8

(b) The minimum exchange ratio acceptable to shareholders of Ajay Limited is :


P2 S1
ER2 =
(PE12) (E1+E2) - P2 S2

9 x 12
= = 0.3
9 (36+12) - 9 x 8

(c) 12 (48) PE12


ER1 = - +
8 240

152
9 x 12
ER2 =
PE12 (48) - 72

Equating ER1 and ER2 and solving for PE12 gives, PE12 = 9
When PE12 = 9
ER1 = ER2 = 0.3
Thus ER1 and ER2 intersect at 0.3

8. The present value of FCF for first seven years is


16.00 14.30 9.7 0
PV(FCF) = - - - +
(1.15) (1.15)2 (1.15)3 (1.15)4

0 10.2 16.7
+ + +
(1.15)5 (1.15)6 (1.15)7

= - Rs.20.4 million
The horizon value at the end of seven years, applying the constant growth model is

FCF8 18
V4 = = = Rs.257.1 million
0.15-0.08 0.15 0.08

1
PV (VH) = 257.1 x = Rs.96.7 million
(1.15)7

The value of the division is :


- 20.4 + 96.7 = Rs.76.3 million

MINICASE

Solution:

153
(a)
Modern Pharma Magnum Drugs Exchange
Ratio
Book value per share 2300 650 65
= Rs.115 = Rs.65
20 10 115
Earnings per share 450 95 9.5
= Rs.22.5 = Rs.9.5
20 10 22.5
Market price per share Rs.320 Rs.102 102

320

Exchange ratio that gives equal weightage to book value per share, earnings per share, and market
price per share

65 9.5 102
+ +
115 22.5 320 0.57 + 0.42 + 0.32
= = 0.44
3 3

(b) An exchange ratio based on earnings per share fails to take into account the
following:

(i) The difference in the growth rate of earnings of the two companies.
(ii) The gains in earnings arising out of merger.
(iii) The differential risk associated with the earnings of the two companies.

(c) Current EPS of Modern Pharma


450
= = Rs.22.5
20

If there is a synergy gain of 5 percent, the post-merger EPS of Modern Pharma is

(450 + 95) (1.05)

20 + ER X 10
Equating this with Rs.22.5, we get

(450 + 95) (1.05)


= 22.5

154
20 + 10ER
This gives ER = 0.54

Thus the maximum exchange ratio Modern Pharma should accept to avoid initial dilution of EPS is
0.54

(d) Post-merger EPS of Modern Pharma if the exchange ratio is 1:4, assuming no
synergy gain:

450 + 95
= Rs.24.2
20 + 0.25 x 10

(e) The maximum exchange ratio acceptable to the shareholders of Modern Pharma if
the P/E ratio of the combined entity is 13 and there is no synergy gain

-S1 (E1 + E2) P/E12


ER1 = +
S2 P1 S2

- 20 (450 + 95) 13
= + = 0.21
10 320 x 10

(f) The minimum exchange ratio acceptable to the shareholders of Magnum Drugs if
the P/E ratio of the combined entity is 12 and the synergy benefit is 2 percent
P2S1
ER2 =
(P/E12) (E1 + E2) (1 + S) P2S2

102 x 20
=
12 (450 + 95) (1.02) 102 X 10
= 0.36

(g) The level of P/E ratio where the lines ER1 and ER2 intersect.

To get this, solve the following for P/E12


- S1 (E1 + E2) P/E12 P2S1
+ =
S2 P1S2 P/E12 (E1 + E2) P2S2

155
- 20 (450 +95) P/E12 102 x 20
+ =
10 320 x 10 P/E12 (450 +95) 1020

- 6400 + 545 P/E12 2040


=
3200 545 P/E12 1020

(545 P/E12 1020) (545 P/E12 6400) = 2040 x 3200

297025 P/E212 3488000 P/E12 555900 P/E12


+6528000 = 6528000
2
297025 P/E 12 = 4043900 P/E
297025 P/E12 = 4043900
P/E12 = 13.61

156
Chapter 37
INTERNATIONAL FINANCIAL MANAGEMENT

1. The annualised premium is :

Forward rate Spot rate 12


x
Spot rate Forward contract length in months

46.50 46.00 12
= x = 4.3%
46.00 3

2. 100
100 (1.06) = x 1.07 x F
1.553

106 x 1.553
F = = 1.538
107
A forward exchange rate of 1.538 dollars per sterling pound will mean indifference between
investing in the U.S and in the U.K.

3. (a) The annual percentage premium of the dollar on the yen may be calculated with
reference to 30-days futures
105.5 105 12
x = 5.7%
105 1

(b) The most likely spot rate 6 months hence will be : 107 yen / dollar

(c) Futures rate 1 + domestic interest rate


=
Spot rate 1 + foreign interest rate

107 1 + domestic interest rate in Japan


=
106 1.03

Domestic interest rate in Japan = .0397 = 3.97 per cent

4. S0 = Rs.46 , rh = 11 per cent , rf = 6 per cent

157
Hence the forecasted spot rates are :
Year Forecasted spot exchange rate
1 Rs.46 (1.11 / 1.06)1 = Rs.48.17
2 Rs.46 (1.11 / 1.06)2 = Rs.50.44
3 Rs.46 (1.11 / 1.06)3 = Rs.52.82
4 Rs.46 (1.11 / 1.06)4 = Rs.55.31
5 Rs.46 (1.11 / 1.06)5 = Rs.57.92

The expected rupee cash flows for the project

Year Cash flow in dollars Expected exchange Cash flow in rupees


(million) rate (million)
0 -200 46 -9200
1 50 48.17 2408.5
2 70 50.44 3530.8
3 90 52.82 4753.8
4 105 55.31 5807.6
5 80 57.92 4633.6

Given a rupee discount rate of 20 per cent, the NPV in rupees is :

2408.5 3530.8 4753.8


NPV = -9200 + + +
(1.18)2 (1.18)3 (1.18)4

5807.6 4633.6
+ +
(1.18)5 (1.18)6

= Rs.3406.2 million

The dollar NPV is :


3406.2 / 46 = 74.05 million dollars

5. Forward rate 1 + domestic interest rate


=
Spot rate 1 + foreign interest rate

F 1 + .015
=
1.60 1 + .020
F = $ 1.592 /

158
6. Expected spot rate a year from now 1 + expected inflation in home country
=
Current spot rate 1 + expected inflation in foreign country

Expected spot rate a year from now 1.06


=
Rs.70 1.03

So, the expected spot rate a year from now is : 72 x (1.06 / 1.03) = Rs.72.04

7. (a) The spot exchange rate of one US dollar should be :


12000
= Rs.48
250
(b) One year forward rate of one US dollar should be :
13000
= Rs.50
260

8. (1 + expected inflation in Japan)2


Expected spot rate = Current spot rate x
2 years from now (1 + expected inflation in UK)2

(1.01)2
= 170 x = 163.46 yen /
(1.03)2

9. (i) Determine the present value of the foreign currency liability (100,000) by using
90-day money market lending rate applicable to the foreign country. This works
out to :
100,000
= 98522
(1.015)
(ii) Obtain 98522 on todays spot market
(iii) Invest 98522 in the UK money market. This investment will grow to
100,000 after 90 days

10. (i) Determine the present value of the foreign currency asset (100,000) by using
the 90-day money market borrowing rate of 2 per cent.
100,000
= 98039
(1.02)

159
(ii) Borrow 98039 in the UK money market and convert them to dollars in the spot
market.

(iii) Repay the borrowing of 98039 which will compound to 100000 after 90 days
with the collection of the receivable

11. A lower interest rate in the Swiss market will be offset by the depreciation of the US
dollar vis--vis the Swiss franc. So Mr.Sehgals argument is not tenable.

160
Chapter 40
CORPORATE RISK MANAGEMENT

1. (a) The investor must short sell Rs.1.43 million (Rs.1 million / 0.70) of B
(b) His hedge ratio is 0.70
(c) To create a zero value hedge he must deposit Rs.0.43 million

2. Futures price Spot price x Dividend yield


= Spot price -
(1+Risk-free rate)0.5 (1+Risk-free rate)0.5

4200 4000 x Dividend yield


= 4000 -
(1.145) 0.5 (1.145) 0.5

The dividend yield on a six months basis is 2 per cent. On an annual basis it is
approximately 4 per cent.

3. Futures price
= Spot price + Present value of Present value
(1+Risk-free rate)1 storage costs of convenience yield

5400
= 5000 + 250 Present value of convenience yield
(1.15)1

Hence the present value of convenience yield is Rs.554.3 per ton.

161
162

163

You might also like